Patient Management Flashcards

1
Q

A patient is sitting in the chair immediately
following an extraction. She says, “Thank you.
That wasn’t as bad as I expected, but my sister
told me that the first night after having a tooth
pulled is very painful. What if the medication
you’re giving me isn’t strong enough?” Choose
the most appropriate response.
A. “Did she make you feel worried about that?”
B. “It sounds like you’re worried that you might not
have enough pain relief when you’re home.”
C. “I understand your concern.”
D. “Don’t worry. I’ll give you plenty of pain
medicine.”
E. “It sounds like your sister had a unusually bad
experience. Don’t believe what others tell you,
and certainly don’t let that worry you. You’ll be
fine.”

A

B. Of the options given, the best response would be

to interpret what the patient is trying to commu-
nicate and reflect the communication back to

him or her. This will gently encourage the patient
to openly express and discuss the concern with

the clinician. It also serves to establish an envi-
ronment of openness and acceptance.

How well did you know this?
1
Not at all
2
3
4
5
Perfectly
2
Q

During admission, a patient interrupts you on
a number of occasions with stories about past
dental experiences while you are attempting to
take a complete medical history. Your best
response would be _____.
A. Say nothing, listen to the patient, and finish your
intake as best you can.
B. Say, “I’d like to focus on your present
experience and right now I need to know your
medical history.”
C. Say, “It seems like you’ve had some important
experiences and I would like to hear more
about them, but first, let’s discuss this health
questionnaire before we address it, okay?”
D. Say, “I don’t need to know the details of your
dental history. Please inform me of the
experiences asked about in the questionnaire.”
E. Say, “We have about 30 minutes to complete
this questionnaire and get started in your
examination, so let’s focus on that.”

A

C. Of the options given, it is best to acknowledge
that the patient is trying to convey information that
is important to him or her and establish that there
will be a time to talk about those issues, while
gently redirecting him or her to the task at hand.

How well did you know this?
1
Not at all
2
3
4
5
Perfectly
3
Q

A 7-year-old child has a history of recurrent
pain and discomfort in a second molar, which
has a necrotic pulp. You present the treatment
options to the parents. “There are several ways
in which we can treat this problem. We could
do a pulpectomy in which we . . . . We could
do something called a pulpotomy, which
involves. . . . We could apply a pulp cap which
is . . . . We could remove the tooth. Or we could
leave the tooth untreated for now and see how
things go.” You have phrased the options so
that they are in what you believe to be the order
of descending desirability and you have
indicated that to the patient. Which option is
most likely to be chosen by the parents?
A. Pulpectomy
B. Pulpotomy
C. Pulp cap
D. Extraction
E. No treatment

A

A. When a number of alternatives are presented
and the first on the list is more desirable, there is
a tendency for individuals to select the first
option and view the successive options as less
desirable.

How well did you know this?
1
Not at all
2
3
4
5
Perfectly
4
Q

Which statement is false regarding motivation?
A. Motivation is strengthened when a person
succeeds and is weakened when a person fails
to achieve his/her goals.
B. Motivation is increased when the patient
focuses on long-term goals.
C. Motivating a patient can be achieved by
generating interest, showing your concern, and
providing information.
D. Encourage a sense of personal acceptance in
the face of the inevitable difficulties involved in
breaking old habits and establishing new ones.
E. Help a patient cope with relapses by
emphasizing the knowledge gained.

A

B. Focusing on long-term goals is not only a poor
motivator, it is often a pitfall in the effort to change
behavior, as patients are less motivated when goals
seem too big, impossible, or far from their current
circumstances.

How well did you know this?
1
Not at all
2
3
4
5
Perfectly
5
Q

Which statement is false about behavioral con-
tracts?

A. It is a legal and binding agreement between
health care professional and patient.
B. It helps solidify an agreement with a patient.
C. It should always be open to modification.
D. It helps clarify agreements.
E. The clinician should give a copy to thepatient
and keep one for himself or herself.

A

A. Although the behavioral contract is not a legal

document, it can be a useful approach in solidi-
fying behavioral strategies and goals.

How well did you know this?
1
Not at all
2
3
4
5
Perfectly
6
Q

A 6-year-old patient likes to tell you stories
about school. Each time he begins a story, you
stop working to listen. After three long sessions,
you realize that the child is attempting to avoid
or delay the dental work by telling stories. You
decide that from this point on you are going to
continue working while engaged in conversation
with the patient. At first, the child tells you more
stories about school and, further, tries other
strategies to get your attention and stop your
work. He eventually settles down and allows
you to work, whether or not you are engaged in
conversation. This is an example of _____.
A. Shaping
B. Extinction
C. Modeling
D. Stimulus control
E. Power

A

B. Extinction is the process of identifying all positive
reinforcements (in this case, the dentist ceasing
work on the child’s teeth) that maintain a behavior
and ceasing or withholding these.

How well did you know this?
1
Not at all
2
3
4
5
Perfectly
7
Q

Which of the following is not a factor in the
appraisal of stress?
A. Familiarity—how familiar the situation is; the
less familiar, the more stressful it may seem.
B. Predictability—how predictable the situation is;
the less predictable, the more stressful it may
seem.
C. Controllability—how controllable the situation
seems to be; the less controllable, the more
stressful it may seem.
D. Imminence—the more imminent the situation
is, the more stressful it may seem.
E. Positive or negative valence—whether the
situation is positive or negative; positive
situations (e.g., a wedding) are typically
experienced as less stressful than are negative
situations (e.g., a divorce).

A

E. Both positive and negative events or situations

are experienced as stress.

How well did you know this?
1
Not at all
2
3
4
5
Perfectly
8
Q

The substitution of a relaxation response for

an anxiety response (using a relaxation strat-
egy such as diaphragmatic breathing) when

one is exposed to a hierarchy of feared stimuli
is called \_\_\_\_\_.
A. Progressive muscle relaxation
B. Habituation
C. Flooding
D. Systematic desensitization
E. Biofeedback
A

D. Systematic desensitization is the process of sys-
tematically pairing a relaxation response with a

hierarchy of feared stimuli.

How well did you know this?
1
Not at all
2
3
4
5
Perfectly
9
Q

Which statement is false regarding the relation-
ship between pain and fear?

A. Fear initially inhibits pain due to a release of
endorphins from the pituitary, resulting in an
analgesic effect.
B. Although muscle tension contributes to the
experience of anxiety, it does not contribute to
the perception of pain.
C. Any autonomic activation causes one to have a
lower pain threshold.
D. Catastrophic thinking and a perceived lack of
control are common factors that influence pain
perceptions.
E. Misattribution occurs when patients identify an
event as painful because they can identify a
fearful stimulus.

A

B. Muscle tension is associated with the experience
of anxiety. Heightened anxiety contributes to

lower pain thresholds/sensitivity to the percep-
tion of pain.

How well did you know this?
1
Not at all
2
3
4
5
Perfectly
10
Q

Which of the following is an example of a cog-
nitive strategy that may be useful in pain man-
agement?

A. Address expectations by providing information
and addressing any questions and/or concerns.
B. Suggest to patients that they learn to identify,
evaluate, and eliminate maladaptive thinking.
C. Encourage patient efforts to address their
anxiety and pain management.
D. Suggest to patients that they learn to generate,
evaluate, and apply more realistic thinking.
E. All of the above.

A

E. All of the strategies listed may be considered

appropriate cognitive interventions in pain man-
agement.

How well did you know this?
1
Not at all
2
3
4
5
Perfectly
11
Q
  1. Which of the following scenarios is an example
    of classical conditioning?
    A. You teach a dentally-anxious patient
    diaphragmatic breathing unconditional stimulus
    (US), which naturally induces the physiological
    relaxation response unconditional response
    (UR). You seat that anxious patient in the dental
    chair for an examination conditional stimulus
    (CS) and ask them to use their breathing skills
    during the exam (US). While using the breathing
    skills, the patient will feel more relaxed
    conditional response (CR).
    B. You teach a dentally-anxious patient
    diaphragmatic breathing (US), which naturally
    induces the physiological relaxation response
    (UR). You ask the patient to practice that
    technique at home (CS) and also use it during
    procedures to reduce the subjective experience
    of anxiety (CR).
    C. You teach a dentally-anxious patient
    diaphragmatic breathing (US), which naturally
    induces the physiological relaxation response
    (UR). You seat that anxious patient in the dental
    chair for an examination (CS) and ask them to
    use their breathing skills during the exam (US).
    The focus on breathing serves as a distraction
    (US) from what the patient feels is threatening
    and fearful (CR), and, therefore reports less
    anxiety (CR).
    D. You teach a dentally-anxious patient
    diaphragmatic breathing (US), which naturally
    induces the physiological relaxation response
    (UR). You seat that anxious patient in the dental
    chair for an examination (CS) and ask them to
    use their breathing skills during the exam (US).
    After a number of these experiences, the patient
    will feel relaxed during the exam while using
    the breathing technique (UR) and without using
    it at all (CR).
    E. None of the above.
A

D. Classical conditioning (also known as respondent
or Pavlovian conditioning) occurs when a neutral
stimulus, one that is not associated with a particular
response, is paired with an unconditioned stimulus
([US]; one that naturally elicits a particular
response [UR]). After a number of pairings, the
neutral stimulus (CS) elicits a conditioned response
(CR), which is essentially a weaker form of the UR)
without the presence of the US.

How well did you know this?
1
Not at all
2
3
4
5
Perfectly
12
Q

The best strategy for addressing dental fear
that is based upon distrust of the dentist is to
_____.
A. Use distraction techniques
B. Use cognitive coping strategies
C. Enhance informational and behavioral control
D. Teach diaphragmatic breathing
E. Reassure the patient that he or she can trust
you

A

C. Providing the patient with information and
control over his or her environment is likely to
contribute to increased trust over time. Avoiding
the issue of trust or providing reassurance that
the patient can trust you without evidence is
likely to maintain poor trust.

How well did you know this?
1
Not at all
2
3
4
5
Perfectly
13
Q

What behavior can you typically expect from
an anxious patient in the dental chair?
A. He or she is more likely to sit still, hands clasped
together.
B. He or she is more likely to sit casually, legs
crossed, reading a magazine.
C. He or she is more likely to keep to himself or
herself and not speak unless spoken to.
D. He or she is more likely to fidget in the chair,
moving his or her hands and feet.
E. Both A and C.

A

E. Contrary to their behavior in the waiting room,
anxious patients are typically more likely to sit

very still, often holding onto the arms of the den-
tal chair, and engage in minimal verbal commu-
nication unless encouraged by the clinician.

How well did you know this?
1
Not at all
2
3
4
5
Perfectly
14
Q

With no other intervention or instruction,
which is most likely to trigger a physiological
relaxation response?
A. Observing one’s own physiological responses
(e.g., heart rate, blood pressure)
B. Muscle tensing
C. Reassurance
D. Thought stopping
E. Diaphragmatic breathing

A

E. Diaphragmatic breathing naturally activates the
parasympathetic nervous system, producing a
relaxation response.

How well did you know this?
1
Not at all
2
3
4
5
Perfectly
15
Q

A 32-year-old male patient is fearful of receiv-
ing injections. You decide to use a cognitive

behavioral strategy with him to help him
through an injection. You have already
instructed him in diaphragmatic breathing and
ask him to practice this skill throughout the
procedure. First, you show him the syringe. You
talk about the characteristics of the needle.
You then place the needle in his mouth with the
cap on. Then, you simulate the procedure with
the cap on. You then simulate the procedure
with the cap off. Eventually, you proceed with

the injection. What does this procedure exem-
plify?

A. Habituation
B. Cognitive control
C. Flooding
D. Systematic desensitization
E. Behavior modification
A

D. Systematic desensitization is the systematic
process of exposing the patient to a hierarchy of
increasingly anxiety-provoking stimuli while the
patient uses relaxation skills such as diaphragmatic
breathing exercises.

How well did you know this?
1
Not at all
2
3
4
5
Perfectly
16
Q

Principles of operant conditioning teach us
that _____.
A. If you praise your 5-year-old patient and reward
him for keeping his legs still while you are
drilling, this will make the child happy and more
likely to like you and less likely to resist your
requests.
B. If you praise your 5-year-old patient and reward
him for keeping his legs still while you are
drilling, this will increase the likelihood that he
will remain still in similar situations in the future.

C. If you make the dental environment a child-
friendly place, your young patient will be more

comfortable.
D. If you pair the dental chair with having a parent
present, the child will be less likely to be
anxious.
E. None of the above.

A

B. Operant conditioning posits that behavior is

largely influenced by the consequences associ-
ated with the particular behavior.

How well did you know this?
1
Not at all
2
3
4
5
Perfectly
17
Q

According to anxiety disorders research, it has
been suggested that which of the following is
the most important component of systematic
desensitization?
A. Cognitive restructuring
B. Progressive muscle relaxation
C. Diaphragmatic breathing
D. Exposure
E. Psychoeducation

A

D. Research suggests that the most integral compo-
nent of the treatment of anxiety is exposure to

the feared stimulus.

How well did you know this?
1
Not at all
2
3
4
5
Perfectly
18
Q

Sarah S. is a young child who consistently pres-
ents as anxious, hypervigilant, and upset during

dental visits. Sarah is often accompanied by her
parent, who appears to be very concerned about
the child and wants to be involved at all times in
her evaluation and treatment. During this visit,
Sarah’s treatment requires an injection and a
rubber dam application, which you anticipate
may lead to increased anxiety. Which strategy
would be the least effective in completing the
rubber dam application?
A. Tell-Show-Do
B. Distraction
C. Ask the child to be a helper
D. Structure time
E. Rehearsals

A

B. Of the choices, distraction would most likely be
the least effective approach—the attention of a
very anxious individual cannot typically be easily
diverted. In such cases, distraction can have

detrimental effects such as compromising rap-
port and/or increasing anxiety by failing to pro-
vide a positive coping experience. Providing

education and coping strategies—increasing pre-
dictability, familiarity, and controllability—are

typically more effective strategies in working with
anxious patients.

How well did you know this?
1
Not at all
2
3
4
5
Perfectly
19
Q

Which of the following factors are involved in
the cognitive appraisal of a threat?
A. Interference, adaptability, longevity, and reactance
B. Adaptability, preventability, inevitability, and
constancy
C. Controllability, familiarity, predictability, and
imminence
D. Validity, reliability, adaptability, and predictability
E. Accountability, reliability, validity, and familiarity

A

C. Controllability, familiarity, predictability, and
imminence are significant factors influencing the
cognitive appraisal of stress.

How well did you know this?
1
Not at all
2
3
4
5
Perfectly
20
Q

A patient has difficulty inhibiting the gag reflex
during x-ray procedures. You suggest that the

patient take several x-ray packets home and pra-
ctice holding the packets in his or her mouth for

increasingly longer periods of time. Which of the
following techniques does this best exemplify?
A. Reinforcement
B. Graded exposure
C. Modeling
D. Behavioral control
E. Systematic desensitization

A

B. Graded exposure is the systematic process of

exposing the patient to a hierarchy of increas-
ingly anxiety-provoking stimuli.

How well did you know this?
1
Not at all
2
3
4
5
Perfectly
21
Q

When faced with a frightened child patient,
which would be the most appropriate or most
effective response?
A. Ask the child about his or her fears.
B. Reschedule the appointment for a later date.
C. Reassure the child.
D. Tell the child that dentistry shouldn’t be frightening.
E. Chastise the child.

A

A. Asking the child about his or her fears will create
an environment in which the child is encouraged
to discuss any worries or concerns and to ask
questions. This will also serve to alleviate anxiety,

provide an opportunity to correct any mispercep-
tions regarding dentistry, and to further establish

or maintain trust and rapport.

How well did you know this?
1
Not at all
2
3
4
5
Perfectly
22
Q

Research suggests that life events and per-
ceived stress/distress _____ predictors of self-
reported health concerns.

A. Are
B. Are not
C. Are sometimes
D. Have little to do with
E. None of the above
A

A. Perceived stress and distress in one’s life has
been demonstrated to be a significant predictor
(positively correlated) with self-reported health
concerns.

How well did you know this?
1
Not at all
2
3
4
5
Perfectly
23
Q

Patients experiencing stress and anxiety typi-
cally require _____ interpersonal distance for

comfortable interaction.
A. Greater
B. Less
C. The same as patients who are not experiencing
stress and anxiety
D. Individualized
E. Behaviorally controlled
A

A. Patients who are experiencing stress and anxiety
typically feel more comfortable in having greater
interpersonal space than they normally would
when not experiencing stress and anxiety.

How well did you know this?
1
Not at all
2
3
4
5
Perfectly
24
Q

Which statement is true about the use of
silence as an interviewing technique?
A. It permits and encourages patient participation.
B. It is a nonverbal technique for showing interest
in the patient.
C. It is a nonverbal technique for encouraging the
patient to speak.
D. It is done by silently attending to the patient,
while maintaining eye contact.
E. All of the above.

A

E. The use of silence can be a useful technique to

encourage patient comment following a state-
ment or question posed to the patient.

How well did you know this?
1
Not at all
2
3
4
5
Perfectly
25
Q

How do people typically respond to stress?
A. Physiologically (fight-or-flight response; i.e.,
autonomic arousal)
B. Cognitively (beliefs of self-efficacy, stress
appraisal)
C. Behaviorally (e.g., disturbed sleep/appetite,
impaired attention, acting out)
D. Emotionally (e.g., anxiety, anger, fear)
E. All of the above

A

E. Individuals respond to stress physiologically,

behaviorally, cognitively, and emotionally.

How well did you know this?
1
Not at all
2
3
4
5
Perfectly
26
Q
Which of the following indices is not reversible?
A. DMFT
B. GI
C. PI
D. OHI-S
E. None of the above
A

A. Periodontal disease (measured by the PI) and
gingival disease, measured by the GI, are
reversible processes. The amount of the debris
and calculus, measured by the OHI-S, can
decrease too. Caries is not a reversible process.

How well did you know this?
1
Not at all
2
3
4
5
Perfectly
27
Q

The recommended level of fluoride for commu-
nity water supply systems in the United States

ranges from \_\_\_\_\_.
A. 0.2–0.5 ppm
B. 0.7–1.2 mL
C. 1.2–1.5 ppm
D. 0.2–0.5 mL
E. 0.7–1.2 ppm
A

E. The recommended level of fluoride for a com-
munity water supply in the United States ranges

from 0.7 to 1.2 ppm of fluoride, depending on the
mean maximum daily air temperature over a

5-year period. Thus, in a warm climate the fluo-
ride level would be lower and in a cold climate it

would be higher. In the United States, most com-
munities are fluoridated at approximately 1 ppm,

which is equivalent to 1.0 mg of fluoride per liter
of water.

How well did you know this?
1
Not at all
2
3
4
5
Perfectly
28
Q

The supplemental fluoride daily dosage sched-
ule for a 5-year-old child who lives in a commu-
nity where the concentration of fluoride in the

drinking water is less than 0.3 ppm is \_\_\_\_\_.
A. 0 mg
B. 0.10 mg
C. 0.25 mg
D. 0.50 mg
E. 1 mg
A

D. Physicians and dentists can help prevent fluorosis
by prescribing dietary fluoride supplements
according to the Supplemental Fluoride Dosage
Schedule recommended by the ADA Council on
Scientific Affairs.

How well did you know this?
1
Not at all
2
3
4
5
Perfectly
29
Q
What type of epidemiology is primarily used in
intervention studies?
A. Descriptive
B. Analytical
C. Observational
D. Experimental
E. None of the above
A

D. Experimental epidemiology is used primarily in

intervention studies. Once etiology for a particu-
lar disease has been determined, the researchers will try to establish the effectiveness of a particu-
lar program of prevention or therapy. Descriptive

epidemiology is used to quantify disease status in
a community. Analytical epidemiology, also
called observational epidemiology, is used to
determine the etiology of a disease.

How well did you know this?
1
Not at all
2
3
4
5
Perfectly
30
Q

A researcher follows a group of individuals in
a population over 10 years to determine who
develops cancer, and then evaluates the factors
that affected the group. What type of study is
this?
A. Cross-sectional
B. Case control
C. Randomized
D. Prospective cohort
E. Retrospective cohort

A

D. In this case, the investigator chooses or defines a

sample of subjects who do not yet have the out-
come of interest: cancer. He or she measures risk

factors in each subject (such as habits that may
predict the subsequent outcome) and follows

these subjects with periodic surveys or examina-
tions to detect the outcome(s) of interest.

How well did you know this?
1
Not at all
2
3
4
5
Perfectly
31
Q

A group of researchers undertook a study to
assess the relationship between squamous cell

carcinoma and chewing tobacco. The resear-
chers determined past exposure records among

subjects who had been diagnosed with the dis-
ease. This type of study was a _____.

A. Clinical trial
B. Community trial
C. Retrospective cohort study
D. Case control study
E. Randomized clinical trial
A

C. In a retrospective cohort study, the investigator
chooses a sample of individuals who have the
outcome of interest (in this case, squamous cell

carcinoma) and then look into the past for possi-
ble variables that may have caused the disease

(e.g., chewing tobacco).

How well did you know this?
1
Not at all
2
3
4
5
Perfectly
32
Q

The following part of a scientific article sum-
marizes the background and focus of the study,

the population sampled, and the experimental
design, findings, and conclusion.
A. Introduction
B. Background
C. Literature review
D. Methods
E. Abstract
A

E. The abstract allows the reader to determine

whether the study is of interest. The abstract usu-
ally appears at the head of the article and is

reproduced in the literature database.

How well did you know this?
1
Not at all
2
3
4
5
Perfectly
33
Q

In this section of a scientific article, the resear-
cher interprets and explains the results obtained.

A. Summary and conclusion
B. Results
C. Discussion
D. Abstract
E. None of the above
A

C. In the results section the researcher describes the
specific findings and actual outcomes of the project
but does not interpret them. The interpretation and
analysis of the results are part of the discussion,
where the researcher attempts to explain his
results.

How well did you know this?
1
Not at all
2
3
4
5
Perfectly
34
Q

The following were the scores for six dental stu-
dents in their Restorative Dentistry exam: 56, 64,

68, 46, 82, 86. Therefore, the median is \_\_\_\_\_.
A. 68
B. 64
C. 67
D. 40
E. 66
A

E. The median is the middle of a distribution: half the
scores are above the median and half are below
the median. The median is less sensitive to
extreme scores than the mean, making it a better
measure than the mean for highly skewed
distributions. For instance, the median income of
a population is usually more informative than the
mean income.
When there is an even number of numbers,

the median is the mean of the two middle num-
bers. Thus, in this case the median is (64 + 68)/2

= 66.

How well did you know this?
1
Not at all
2
3
4
5
Perfectly
35
Q

A correlation analysis shows that as the income

of the population increases, the number of deca-
yed teeth decreases. Therefore, an expected value

for this correlation coefficient (r) would be \_\_\_\_\_.
A. 0
B. 1
C. −1
D. 2
E. −2
A

C. The correlation coefficient (r) quantifies the rela-
tionship between variables (x and y). A positive

correlation coefficient indicates that the variables

increase in the same direction; a negative correla-
tion coefficient indicates that the variables vary in

opposite directions. The correlation coefficient
ranges from −1 to +1.

How well did you know this?
1
Not at all
2
3
4
5
Perfectly
36
Q

A test result that erroneously excludes an indi-
vidual from a specific diagnostic or reference

group is called \_\_\_\_\_.
A. Erroneous
B. False positive
C. False negative
D. Mistaken
E. None of the above
A

C. A false positive test is a test result which erro-
neously assigns an individual to a specific diag-
nostic or reference group.

How well did you know this?
1
Not at all
2
3
4
5
Perfectly
37
Q

Which of the following statements about trans-
missible diseases is false?

A. The risk of transmission after percutaneous
injury is higher for HBV than for HIV.
B. HCV and HIV are both caused by an RNA virus.
C. A vaccine to immunize against HBV is available.
D. The average risk of infection for HBV after a
needlestick injury falls between HCV and HIV.
E. All of the above.

A

D. The average risk of infection for HBV after a
needlestick injury does not fall between HCV and
HIV. For HBV, the risk of transmission after
percutaneous injury is 30%; this figure is 1.8% for
HCV and 0.3% for HIV.

How well did you know this?
1
Not at all
2
3
4
5
Perfectly
38
Q

In HIV diagnosis, the Western blot assay is
used to confirm the results of a positive ELISA
test. Therefore, we can say that the Western
blot test will confirm a _____.
A. True-positive result
B. True-negative result
C. False-positive result
D. False-negative result
E. None of the above

A

A. Very specific tests are appropriate for confirming
the existence of a disease. If the result of a highly

specific test is positive, the disease is almost cer-
tain. High specificity is required in situations where the consequences of a false-positive
diagnosis are serious or unduly alarming (e.g.,
HIV positivity).

How well did you know this?
1
Not at all
2
3
4
5
Perfectly
39
Q

Which of the following statement(s) about the
hepatitis B vaccination is(are) true?
A. HBV vaccine must be offered to all potentially
exposed dental workers.
B. The HBV vaccine must be free to all potentially
exposed dental workers.
C. At the time of employment, each person should
be asked to provide documentation of previous
immunizations.
D. Three doses are given to confer immunity.
E. All of the above.

A

E. All of these measures help ensure the safety of

dental personnel.

How well did you know this?
1
Not at all
2
3
4
5
Perfectly
40
Q

Which of the following terms refers specifically
to the process where an antimicrobial agent
destroys (germicide) or avoids the growth
(microbiostatic) of pathogenic microorganisms
on inanimate surfaces?
A. Antisepsis
B. Microbacterial control
C. Sterilization
D. Disinfection
E. Asepsis

A

D. Disinfection refers only to the inhibition or destru-
ction of pathogens. Spores are not killed during

disinfection procedures. By custom, the term
disinfection is reserved for chemicals applied to
inanimate surfaces, and the word antiseptic is
used for antimicrobial agents that are applied to
living tissues.

How well did you know this?
1
Not at all
2
3
4
5
Perfectly
41
Q
Which of the following is the most common
method of sterilization?
A. Dry heat
B. Ethylene oxide
C. Glutaraldehyde at 2%
D. Autoclave
E. Chemi-clave
A

D. The proper time and temperature for autoclaving is
250 ̊ F (121 ̊ C) for 15 to 20 minutes, which
yields 15 pounds pressure of steam, or 270 ̊ F
(134 ̊ C) for a minimum of 3 minutes, which
yields 30 pounds pressure of steam. Moist heat

destroys bacteria—denaturation of the high-
protein-containing bacteria.

How well did you know this?
1
Not at all
2
3
4
5
Perfectly
42
Q

A set of precautions designed to prevent trans-
mission of HIV, HBV, and other bloodborne

pathogens when providing first aid or health
care is known as \_\_\_\_\_.
A. Asepsis
B. Infection control
C. Sterilization
D. Disinfection
E. Standard infection control procedures
A

E. A thorough medical history, physical examination,
and laboratory tests will not always detect patients
who are carriers of infectious diseases. Therefore,
you must assume that all patients are infected with
HIV, HBV, or other bloodborne pathogens. Similar
infection control procedures must be used for all
patients, regardless of their medical history or the
type of treatment to be performed.

How well did you know this?
1
Not at all
2
3
4
5
Perfectly
43
Q
Which of the following chemical agents is not a
disinfectant?
A. Iodophors
B. Sodium hypochlorite
C. Synthetic phenol
D. Isopropyl alcohol
E. Glutaraldehyde
A

D. Alcohol is not an accepted disinfectant. Alcohol

evaporates too quickly to be an effective disin-
fectant. The term disinfection is reserved for

chemicals applied to inanimate surfaces, and
the word antiseptic is used for antimicrobial agents
(such as alcohol) that are applied to living tissues.

How well did you know this?
1
Not at all
2
3
4
5
Perfectly
44
Q

Which of the following recommendations must
be followed when handling mercury?
A. Train personnel involved in the handling of
mercury
B. Work in properly ventilated areas
C. Use high-volume evacuation systems when
finishing or removing amalgams
D. Avoid direct skin contact with the metal
E. All of the above

A

E. Mercury can be absorbed through the skin as
well as absorbed by inhalation. Safe handling,
resulting in part from proper training, helps
reduce the risk of exposure.

How well did you know this?
1
Not at all
2
3
4
5
Perfectly
45
Q

According to the CDC, the acceptable water
quality in a dental office should be _____.
A. < 125 CFU/mL
B. < 250 CFU/mL
C. < 500 CFU/mL
D. < 750 CFU/mL
E. < 1000 CFU/mL

A

C. The CDC recommends, at a minimum to meet
nationally recognized drinking water standards,

less than 500 colony-forming units (CFUs) of het-
erotrophic bacteria per milliliter. In 1995, the ADA

addressed the dental water concern by asking
manufacturers to provide equipment with the
ability to deliver treatment water with < 200
CFU/ml of unfiltered output from waterlines.

How well did you know this?
1
Not at all
2
3
4
5
Perfectly
46
Q

Which of the following ADA’s Principles of
Ethics states that a dentist has a duty to
respect the patient’s right to self-determination
and confidentiality?
A. Patient Autonomy
B. Nonmaleficence
C. Beneficence
D. Justice
E. Veracity

A

A. There are five principles in the ADA Principles of
Ethics:
● Patient Autonomy (“self-governance”). The
dentist has a duty to respect the patient’s rights to
self-determination and confidentiality.
● Nonmaleficence (“do no harm”). The dentist
has a duty to refrain from harming the patient.
● Beneficence (“do good”). The dentist has a
duty to promote the patient’s welfare.
● Justice (“fairness”). The dentist has a duty to
treat people fairly.
● Veracity (“truthfulness”). The dentist has a duty
to communicate truthfully.

How well did you know this?
1
Not at all
2
3
4
5
Perfectly
47
Q
Which of the following are characteristics of
proper documentation in a dental record?
A. Specific
B. Objective
C. Complete
D. Timely
E. All of the above
A

E. Being specific helps to avoid misinterpretation of
reports. Being objective provides the basis for

accuracy in describing events. Being complete
provides the basis for a thorough review of the facts
when reviewing the report. Being timely ensures
the best opportunity to recall all relevant events.

How well did you know this?
1
Not at all
2
3
4
5
Perfectly
48
Q

Which of the following is an arrangement
between a plan and a group of dentists
whereby the providers agree to accept certain
payments (usually less than their usual fees) in
anticipation of a higher volume of patients?
A. PPO
B. Capitation
C. HMO
D. IPA
E. None of the above

A

A. Capitation is a payment mechanism whereby the
dentist is paid a fixed amount irrespective of the
number of patients seen or services provided.
Health Maintenance Organizations (HMOs) are

also called capitation plans because of the pay-
ment mechanism they use. An Individual Practice

Association is a type of plan that combines the risk
of capitation with fee for service reimbursement.

How well did you know this?
1
Not at all
2
3
4
5
Perfectly
49
Q

Which of the following agencies monitors and

prevents disease outbreaks, implements dis-
ease prevention strategies, and maintains

national health statistics?
A. CDC
B. FDA
C. DEA
D. IHS
E. None of the above
A

A. CDC is correct. The U.S. Food and Drug
Administration (FDA) is responsible for protecting
the health of the nation against impure and unsafe
foods, drugs, cosmetics, and other potential
hazards. The Drug Enforcement Administration
(DEA) determines the levels of controlled
substances that have abuse potential. The Indian
Health Services (IHS) focuses on the goal of raising
the health status of Native Americans and Native
Alaskans.

How well did you know this?
1
Not at all
2
3
4
5
Perfectly
50
Q

Which of the following federal agencies is the

U.S. government’s principal agency for protect-
ing the health of all Americans and providing

essential human services?
A. DHHS
B. NIH
C. HRSA
D. AHRQ
E. None of the above
A

A. The DHHS is the U.S. government’s principal
agency for protecting the health of all Americans
and providing essential human services. DHHS

includes 11 agencies and more than 300 pro-
grams. The agencies listed in the answers are

part of the DHHS. The National Institutes of
Health (NIH) is the world’s premier medical
research organization. The Health Resources and
Services Administration (HRSA) provides access
to essential health care services for people who
are low-income, uninsured, or who live in rural
areas or urban neighborhoods where health care
is scarce. The Agency for Healthcare Research
and Quality (AHRQ) supports research on health
care systems, health care quality and cost issues,

access to health care, and effectiveness of med-
ical treatments.

How well did you know this?
1
Not at all
2
3
4
5
Perfectly
51
Q

Empathic understanding reflects which of the
following characteristics?
A. It accurately reflects others’ feelings.
B. It connects the feelings to concrete circumstances
that are likely to be causing the feelings.
C. It accepts the feelings as real and important.
D. It is nonjudgmental: it does not compromise the
listener’s objectivity.
E. All of the above.

A

E. The use of empathy serves a number of purposes,

including all of the choices given.

How well did you know this?
1
Not at all
2
3
4
5
Perfectly
52
Q

A 14-year-old male patient has significant plaque
build-up and one cavity. Upon inquiry, the
patient tells you that he brushes his teeth about
once a day and does not floss because it is
difficult and too time-consuming. Which of the
following would likely be the least effective way
to address his oral hygiene practices and get
him to improve his oral self-care?
A. Educate him regarding the ways in which he can
improve his oral hygiene and avoid dental
problems in the future.
B. Use a collaborative relationship to arrange for
modification of consequences.
C. Use the Premack principle.
D. Set up a behavioral contract.
E. Set up a system of positive reinforcers.

A

A. Although education is an essential component of
patient care overall, research indicates that, in an
effort to influence behavior change, education
alone is not nearly as effective as behavioral
intervention.

How well did you know this?
1
Not at all
2
3
4
5
Perfectly
53
Q

A 20-year-old woman has significant plaque
build-up. Upon inquiry, she tells you that she
brushes twice daily and flosses daily. You
determine that the patient should be educated
about optimal brushing procedures. Which
strategy might you first use in a series of steps
for improving her brushing skills?
A. Demonstrate your recommended brushing
practices.
B. Explain to her good brushing technique.
C. Tell her you don’t believe she brushes twice daily.
D. Ask her to demonstrate her teeth brushing.
E. Clean her teeth.

A

D. In order to initiate behavior change, one must
first fully assess the behavior. In this case, in

order to address the patient’s inadequate brush-
ing technique one should observe the patient

engaging in the behavior in order to identify the
strengths and weaknesses of the process.

How well did you know this?
1
Not at all
2
3
4
5
Perfectly
54
Q

Which of the following statements is false
regarding behavior change?
A. Goals are long-term targets, whereas objectives
are reachable steps/goals along the way.
B. Shaping is a behavior change strategy in which
the patient learns though the dental professional’s
demonstration of the desired behavior.
C. The basic behavioral model consists of
antecedents, behaviors, and consequences.
D. Some consequences will strengthen a behavior
whereas others will weaken it.
E. Generally, the consequences of today’s behavior
will affect the way in which we behave tomorrow.

A

B. This is an example of modeling. Shaping is the
process of achieving successive approximations
toward a desired behavior.

How well did you know this?
1
Not at all
2
3
4
5
Perfectly
55
Q

You need to inject a local anesthetic for a
10-year-old patient. You note that this patient
appears to be very anxious and frequently asks
what you are doing or are about to do. To which
technique is the patient least likely to respond
well?
A. Distraction.
B. Taking a few deep breaths.
C. Bringing his mother in the room for reassurance.
D. Provide him with age-appropriate information
about the injection.
E. Give the patient an amount of time (how long it
will take) and ask him to count.

A

A. Distraction is not typically a very effective tech-

nique for very anxious patients.

How well did you know this?
1
Not at all
2
3
4
5
Perfectly
56
Q

Which statement is false regarding child pain
management?
A. It is recommended that a dentist provide specific
direction and praise for cooperation.
B. Children do not have a fully developed sense of
time; therefore, it is recommended that one use
more concrete measures of time such as counting
or a visible timer (e.g., egg timer).
C. The Tell-Show-Do technique is no longer
recommended since it has been demonstrated to
increase anxiety and reports of pain during
treatments.
D. In order to enhance the sense of control, it is
recommended that one establish a hand signal
signifying distress and a desire for the dentist to
stop or take a break.
E. Once a child begins to complain about poor pain
control, the chances of having a successful visit
drop dramatically.

A

C. The Tell-Show-Do method, in which the clinician
explains, demonstrates, and allows a child (or an
adult patient) to learn and understand what will
be happening before proceeding, contributes to
decreased self-reports of anxiety and pain.

How well did you know this?
1
Not at all
2
3
4
5
Perfectly
57
Q

What behavior can you typically expect from an
anxious patient in the waiting room?
A. He or she is more likely to sit still, hands clasped
together.
B. He or she is more likely to sit casually, legs
crossed, reading a magazine.
C. He or she is more likely to keep to himself or
herself and not speak unless spoken to.
D. He or she is more likely to fidget in the chair,
moving his or her hands and feet.
E. Both A and C.

A

D. Anxious patients are typically more likely to fidget
in their chairs, unable to focus on a task such as
reading or relaxing in the waiting room.

How well did you know this?
1
Not at all
2
3
4
5
Perfectly
58
Q

What is the most likely consequence of the
avoidance of a feared stimulus?
A. Reinforcement of the associated anxiety.
B. Habituation to the stimulus.
C. Decreased anxiety in response to the stimulus.
D. Learned helplessness.
E. Increased coping resources.

A

A. Avoidance of a feared stimulus inadvertently rein-
forces the anxiety reaction, thereby maintaining

the associated anxiety.

How well did you know this?
1
Not at all
2
3
4
5
Perfectly
59
Q

The first time you perform a complicated dental
procedure, you feel uncomfortable and nervous.
At one point, you even think for a moment that
you will not be able to complete the procedure.
However, you stay with it, and near the end of
the procedure you feel much better. Which
concept does this best exemplify?
A. Covert conditioning
B. Systematic desensitization
C. Habituation
D. Cognitive restructuring
E. Psychoeducation

A

C. Habituation is the decrease in response that

occurs as a result of repeated or prolonged expo-
sure to a conditioned stimulus.

How well did you know this?
1
Not at all
2
3
4
5
Perfectly
60
Q

During a previous dental visit, you assisted a
patient by generating his statement, “Even if
there is some pain, it will be brief. I have ways
to cope and I’ve done well using them.” The
patient will remind himself of this during future
dental procedures. This patient’s statement
exemplifies which of the following strategies?
A. Rational response
B. Self-efficacy induction
C. Relaxation statement
D. Imagery
E. Systematic desensitization

A

A. A rational response is a cognitive therapy tech-
nique in which the patient develops (with or

without assistance) a more adaptive thought or
statement as a means of coping.

How well did you know this?
1
Not at all
2
3
4
5
Perfectly
61
Q

In clinical practice, you frequently see young
patients who are nervous about seeing the
dentist. Knowing which factors are important
influences on young patients’ comfort, you
consider which of the following to help your
patients to feel more comfortable?
A. Inviting a parent into the operatory for support.
B. Placing toys and children’s books in the waiting
room.
C. Hanging child-friendly décor in the operatory.
D. Talking to the child about his or her interests
before beginning your work.
E. All of the above.

A

E. Creating a child-oriented environment (e.g., hav-
ing toys and books in the waiting room, hanging

pictures on the wall and/or ceiling that a child

would find interesting), conveying interest in the
child by asking about their interests, and having
the parent present are all variables that may put
child patients more at ease.

How well did you know this?
1
Not at all
2
3
4
5
Perfectly
62
Q
Dental intervention studies suggest that
educating patients regarding dental care
(patient education) is more effective than
behavioral modification (behavioral
intervention) in increasing compliance.
A. True.
B. False.
C. Sometimes.
D. Both are equally effective.
E. Cannot be determined.
A

B. Research has demonstrated that behavioral inter-
vention is typically more effective than patient

education alone. A combination of the two is
considered the most effective approach to
increasing patient compliance.

How well did you know this?
1
Not at all
2
3
4
5
Perfectly
63
Q

Which technique is typically not useful in
treating the anxious patient?
A. Using less structure in establishing rapport.
B. Reassuring the patient by telling the patient not to
worry.
C. Providing reasons before asking for sensitive
information.
D. Using empathy.
E. Making expectations clear.

A

B. Clinicians should use caution in providing pre-
mature reassurance because, if the outcomes

are inconsistent with what the clinician asserted,
trust and rapport may be compromised.

How well did you know this?
1
Not at all
2
3
4
5
Perfectly
64
Q
The most common site for oral cancers in the
oral cavity is \_\_\_\_\_.
A. Lip
B. Soft palate
C. Hard palate
D. Tongue
E. Tonsils
A

D. The tongue is the most common place for inci-

dent cancers in the oral cavity.

How well did you know this?
1
Not at all
2
3
4
5
Perfectly
65
Q
The most effective method to prevent caries on
the occlusal surfaces among school-age children
is \_\_\_\_\_.
A. Sealants
B. Community water fluoridation
C. School dietary fluoride
D. School fluoride mouth rinse
E. School fluoridation
A

A. Sealants. Community water fluoridation is the
most cost-effective and economical method to
prevent dental caries. However, fluoride is
believed to be the least effective on the occlusal
surface. Most decay among school children
occurs on the chewing surfaces’ pits and surfaces.

How well did you know this?
1
Not at all
2
3
4
5
Perfectly
66
Q
In this type of study design, neither the subject
nor the investigator knows to which group a
subject belongs.
A. Matching studies
B. Randomized
C. Double-blind
D. Single-blind
E. None of the above
A

C. Double-blind designs help prevent the potential
for a biased interpretation of a treatment effect

that might occur if either the investigator or sub-
jects know to which group the latter belong.

How well did you know this?
1
Not at all
2
3
4
5
Perfectly
67
Q
The following component of a scientific article
provides the reader with detailed information
regarding the study design.
A. Introduction
B. Background
C. Literature review
D. Methods
E. Abstract
A

D. The Methods section organizes the research
paper and allows the reader to assess the validity
of the study and the reliability of the measures.

This section should provide the reader with spe-
cific and detailed information regarding how the

study was conducted. Based on this information,
the reader should be able to replicate the study.

How well did you know this?
1
Not at all
2
3
4
5
Perfectly
68
Q

The variance for data set A is 25 and for data
set B is 9. Therefore, we can conclude _____.
A. There are more items in data set A than data set B
B. The mean of data set B is smaller than the mean
for data set A
C. The items in data set A are more widely spread
about the mean value than in data set B
D. The standard deviation for data set B is larger
than for data set A
E. None of the above

A

C. The variance determines the way individual val-
ues are located around the mean. The larger

the variance, the more widely the data items
are spread about the mean value. Variance is
measured in squared units (s2).
The standard deviation is the square root of the
variance. The mean is expressed in the same units as the data items, but the variance is
expressed in squared units. Thus, the standard
deviation measures the average deviation from
the mean in the same units as the mean.

How well did you know this?
1
Not at all
2
3
4
5
Perfectly
69
Q
What route of transmission is a needlestick
injury of infectious disease?
A. Direct contact
B. Indirect contact
C. Accidental contact
D. Parenteral contact
E. Droplets
A

D. Parenteral contact is defined as the transmission
of pathogenic microorganisms by piercing the
skin or mucous membrane (e.g., intravenous,
subcutaneous, intramuscular) by an accidental
or intentional stick with a needle or other sharp
instrument that is contaminated with blood or
other body fluid.

How well did you know this?
1
Not at all
2
3
4
5
Perfectly
70
Q

Which of the following is/are recommendations
for the use of masks?
A. Use whenever aerosols or spatter may be
generated.
B. A new mask should be worn for each patient.
C. Masks should be changed at least once every
hour.
D. Masks should be changed more frequently in the
presence of heavy aerosol contamination.
E. All of the above.

A

E. Masks that cover the mouth and nose reduce

inhalation of potentially infectious aerosol parti-
cles. They also protect the mucous membranes

of the mouth and nose from direct contamina-
tion. Masks should be worn whenever aerosols

or spatter may be generated.
If a mask is worn longer than 20 minutes in an
aerosol environment, the outside surface of the
mask becomes a nidus of pathogenic bacteria
rather than a barrier. It is recommended that a
new mask be worn for each patient and that
masks be changed routinely at least once every
hour and more often in the presence of heavy
aerosol contamination.

How well did you know this?
1
Not at all
2
3
4
5
Perfectly
71
Q

The following definition refers specifically to the
process in which an antimicrobial agent
destroys (germicide) or inhibits the growth
(microbiostatic) of pathogenic microorganisms
on inanimate surfaces.
A. Antisepsis
B. Microbacterial control
C. Sterilization
D. Disinfection
E. Asepsis

A

D. Disinfection refers only to the inhibition or
destruction of pathogens. Spores are not killed
during disinfection procedures. By custom, the
term disinfection is reserved for chemicals

applied to inanimate surfaces, and the term anti-
septic is used for antimicrobial agents that are

applied to living tissues.

How well did you know this?
1
Not at all
2
3
4
5
Perfectly
72
Q

The following biological test is used to check the
effectiveness of the sterilization process.
A. Spore test
B. Total bacterial count test
C. Aseptic test
D. EPA test
E. Disinfection test

A

A. The spore test is a biological monitor. The
process consists of placing into the autoclave
bacterial spores on strips or in envelopes along
with a normal instrument load. If the autoclave is

working properly, the autoclave reaches the tem-
perature and pressure to kill the spores. Spore

testing must be conducted weekly.

How well did you know this?
1
Not at all
2
3
4
5
Perfectly
73
Q
Which of the following are guidelines for
disinfectants used in dental practice?
A. Have an EPA registration number.
B. Kill the Mycobacterium tuberculosis.
C. Have an ADA seal of approval.
D. Must be used according to guidelines.
E. All of the above.
A

E. A disinfectant should be able to kill the

Mycobacterium tuberculosis. This is the bench-
mark organism for disinfectants. It is much

harder to kill than most bacteria, viruses, fungi,
and protozoa. This resistance is partially due to
the waxy cell wall of Mycobacterium.

How well did you know this?
1
Not at all
2
3
4
5
Perfectly
74
Q

Which of the following statements about material
safety data sheets (MSDSs) is/are correct?

A. Employees have the right to know about on-
the-job hazards.

B. The MSDSs help to protect employees.
C. An MSDS contains information on hazardous
materials, substance, and wastes.
D. The MSDS describes chemical hazards and how
to work with the chemical safely.
E. All of the above.

A

E. The Material Safety Data Sheet (MSDS) is an easy

reference for information on hazardous sub-
stances. The MSDS must be “readily accessible”

to workers exposed to hazardous substances.
The MSDS provides information on hazardous
materials, substances, and wastes. Chemical
manufacturers develop and provide an MSDS for
each hazardous product. The distributor is
responsible for getting MSDSs to employers. At
least one copy of the MSDS should be maintained
with the chemical.

How well did you know this?
1
Not at all
2
3
4
5
Perfectly
75
Q

Some dental plans allow the dentist to charge
the patient any difference between what the plan
agrees to pay and the dentist’s UCR (usual,
customary, reasonable) fees. This arrangement
is called _____.
A. Payment differential
B. Balance billing
C. Prospective reimbursement
D. Managed care
E. None of the above

A

B. Balance billing. Prospective reimbursement is a

mechanism in which the dentist is compen-
sated before treatment is provided (i.e., in capi-
tation systems). Managed Care is an

arrangement in which a third party mediates
between providers and patients negotiating

reimbursement for certain services and over-
seeing the treatments delivered.

How well did you know this?
1
Not at all
2
3
4
5
Perfectly
76
Q

when is recommended to fluoridate public water?

A

when levels < 0.7 mg/dL

How well did you know this?
1
Not at all
2
3
4
5
Perfectly
77
Q

what can fluorine inhibit

A

enzyme phosphatase at 20-40mg/day (Ca metabolism)

How well did you know this?
1
Not at all
2
3
4
5
Perfectly
78
Q

what can fluorine do at higher doses

A

40-70 heartburn and pain in extremities

How well did you know this?
1
Not at all
2
3
4
5
Perfectly
79
Q

tx for F toxicity

A

Ca

How well did you know this?
1
Not at all
2
3
4
5
Perfectly
80
Q

which surfaces benefit most from F

A

proximal and smooth

How well did you know this?
1
Not at all
2
3
4
5
Perfectly
81
Q

gretatest concentration of F

A

outer enamel

How well did you know this?
1
Not at all
2
3
4
5
Perfectly
82
Q

concentration of F in school water

A

~1ppm (4.5x city level)

How well did you know this?
1
Not at all
2
3
4
5
Perfectly
83
Q

excretion of F

A

kidney, urine, sweat, up to 3mg/day

How well did you know this?
1
Not at all
2
3
4
5
Perfectly
84
Q

optimal water F

A

0.7-1.2 ppm

How well did you know this?
1
Not at all
2
3
4
5
Perfectly
85
Q

cariostatic effect of F at which stage

A

calcification

How well did you know this?
1
Not at all
2
3
4
5
Perfectly
86
Q

toothpaste F concentration

A

1100 ppm

How well did you know this?
1
Not at all
2
3
4
5
Perfectly
87
Q

F toxicity

A

adults 4-5g

child lethal dose 15 mg/kg

How well did you know this?
1
Not at all
2
3
4
5
Perfectly
88
Q

How many mg of fluoride in 1 L of water at 1 ppm:

A

1 mg

How well did you know this?
1
Not at all
2
3
4
5
Perfectly
89
Q

Patient has 1 ppm fluoride in water. What is that equal to in mg/L?

A

1mg/L

How well did you know this?
1
Not at all
2
3
4
5
Perfectly
90
Q

What ion gets replaced in hydroxyapatite by fluoride?

A

Hydroxyl

How well did you know this?
1
Not at all
2
3
4
5
Perfectly
91
Q

least soluble -

A

fluorapatite

How well did you know this?
1
Not at all
2
3
4
5
Perfectly
92
Q

F- on collagen

A

breaks it down
is bacteriocidal, fluoroapetite is more resistant to acid attack, decreases solubility of enamel,
excreted by kidneys, helps remineralize

How well did you know this?
1
Not at all
2
3
4
5
Perfectly
93
Q

Fluoride helps prevent caries in all ways except?

A

lower pH of the oral cavity

How well did you know this?
1
Not at all
2
3
4
5
Perfectly
94
Q

Where does fluoride localize/accumulates?

A

Outer enamel

How well did you know this?
1
Not at all
2
3
4
5
Perfectly
95
Q

Fluorosis does what?

A

Inhibits remineralization

How well did you know this?
1
Not at all
2
3
4
5
Perfectly
96
Q

Fluorosis does what –

A
inhibits remineralization (irreversible)
- Fluoride induces enamel hypoplasia (hypocalification), which is a characteristic of fluorosis that is caries resistant.
How well did you know this?
1
Not at all
2
3
4
5
Perfectly
97
Q

fluoride on dental plaque

A

no effect

How well did you know this?
1
Not at all
2
3
4
5
Perfectly
98
Q

How do you determine the severity of fluorosis?

A

Look at the two worst teeth?

- Higher the fluoride level, greater degree of enamel change

How well did you know this?
1
Not at all
2
3
4
5
Perfectly
99
Q

ADA recommends to apply in-office fluoride foam for how long?

A

4 minutes

How well did you know this?
1
Not at all
2
3
4
5
Perfectly
100
Q

How many minutes do you place neutral sodium fluoride tray on teeth?

A

4 minutes

How well did you know this?
1
Not at all
2
3
4
5
Perfectly
101
Q

Floride supplementation is effective in: everybody, only kids, anyone but most beneficial to children.

A

anyone but most beneficial to children.

How well did you know this?
1
Not at all
2
3
4
5
Perfectly
102
Q

At what age should supplemental fluoride be started?

A

6 months

How well did you know this?
1
Not at all
2
3
4
5
Perfectly
103
Q

Minimum fluoride age?

A

6 months

How well did you know this?
1
Not at all
2
3
4
5
Perfectly
104
Q

What age does fluoride get incorporate into primary dentition?

A

4 months in utero

How well did you know this?
1
Not at all
2
3
4
5
Perfectly
105
Q

At what age does fluorosis of anterior permanent teeth occur? 4-6mo, 0-4mo, 1year, 2years and 6 years)

A

4-6mo

How well did you know this?
1
Not at all
2
3
4
5
Perfectly
106
Q

Fluoride is given to children in schools usually by rinse with what concentration? 0.05 daily, 0.2 daily, 0.05 weekly, 0.2 weekly

A

0.2 weekly

How well did you know this?
1
Not at all
2
3
4
5
Perfectly
107
Q

How do they administer Fluoride in schools?

A

0.2% Fluoride rinse 1x week

How well did you know this?
1
Not at all
2
3
4
5
Perfectly
108
Q

What happens when a kid with primary teeth ingests fluoride?

A

It affects their permanent teeth.

How well did you know this?
1
Not at all
2
3
4
5
Perfectly
109
Q

The drinking water supply of a community has a natural F level of 0.6 ppm. The F level is raised by 0.4ppm. Tooth decay is expected
to decrease by what % after 7 years?

A

40%

How well did you know this?
1
Not at all
2
3
4
5
Perfectly
110
Q

The usual metabolic path of ingested fluoride primarily involves urinary excretion with remaining portion in?

A

skeletal tissue

How well did you know this?
1
Not at all
2
3
4
5
Perfectly
111
Q

Where is the biggest storage of fluoride in tissues?

A

Skeletal tissues

How well did you know this?
1
Not at all
2
3
4
5
Perfectly
112
Q

Where does fluoride work the best?

A

interproximal and smooth surfaces

How well did you know this?
1
Not at all
2
3
4
5
Perfectly
113
Q

What is least likely to cause baby bottle caries?

a. Breast milk at night
b. Formula made with fluoridated water
c. Water with no fluoride
d. juice

A

c. Water with no fluoride

How well did you know this?
1
Not at all
2
3
4
5
Perfectly
114
Q

Early Childhood Caries (ECC) are cause by all at night except?

  • bottle feeding with formula with fluoridated water
  • breast feeding
  • sippy cup with OJ
  • bottle feeding with processed water with no fluoride
A

bottle feeding with processed water with no fluoride

How well did you know this?
1
Not at all
2
3
4
5
Perfectly
115
Q

ECC (early childhood caries) is usually in what location?

a. Max incisors and molars
b. Man incisor and molars
c. Max canine
d. Man canine and molar

A

a. Max incisors and molars

- Primary max incisors (B&L), then primary molars, mandibular unaffected bc tongue blocks

How well did you know this?
1
Not at all
2
3
4
5
Perfectly
116
Q

What determines fluoride supplementation for a city -

A

temperature

How well did you know this?
1
Not at all
2
3
4
5
Perfectly
117
Q

Usual/recommended water fluoridation-

A

0.7 ppm

How well did you know this?
1
Not at all
2
3
4
5
Perfectly
118
Q

The appropriate/optimal amount of fluoride in the community water:

A

0.75-1.2 ppm

How well did you know this?
1
Not at all
2
3
4
5
Perfectly
119
Q

What percentage of Americans have public fluoride in water: 66%, 85%,

A

66

- CDC 2010 reports Americans have 79.6% water fluoridation

How well did you know this?
1
Not at all
2
3
4
5
Perfectly
120
Q

What is percentage of community water fluoridation- 67, 85, 35

A

67

How well did you know this?
1
Not at all
2
3
4
5
Perfectly
121
Q

Fluoridation for water: effectiveness: early studies showed that it prevents 50%-70% of caries in permanent teeth, however
currently the effectiveness is

A

20%-40%

How well did you know this?
1
Not at all
2
3
4
5
Perfectly
122
Q

Effectiveness of Water fluoridation in the U.S. is

A

20%-40%

How well did you know this?
1
Not at all
2
3
4
5
Perfectly
123
Q

Fluoridation: daily use of tablet cause __ in new carious lesions

A

30% reduction

How well did you know this?
1
Not at all
2
3
4
5
Perfectly
124
Q

Pt has a white discoloration with no sensitivity near cervical region of #29, what do you do? fill, 5% fluoride, do nothing

A

5% fluoride

How well did you know this?
1
Not at all
2
3
4
5
Perfectly
125
Q

What type of fluoride is in water? fluorosilicic acid, sodium fluorosilicate, and sodium
fluoride

A

fluorosilicic acid,

(hydrofluorosilicate) – most commonly used

How well did you know this?
1
Not at all
2
3
4
5
Perfectly
126
Q

Types of Fluoride used in toothpaste: sodium fluoride, Stannous fluoride, sodium monofluorophosphate

A
Stannous fluoride (most effective)
- Stannous fluoride may stain.
How well did you know this?
1
Not at all
2
3
4
5
Perfectly
127
Q

What mouthwash is good for children with caries to rinse with?

A

Sodium Fluoride (NaF)

How well did you know this?
1
Not at all
2
3
4
5
Perfectly
128
Q

What rinse is used at home for developmental disabled child to reduce of plaque? NaF, stannous fluoride, chlorohexidine

A

chlorohexidine

How well did you know this?
1
Not at all
2
3
4
5
Perfectly
129
Q

Which type of fluoride is not in toothpaste?

A

Acidulated fluoride

How well did you know this?
1
Not at all
2
3
4
5
Perfectly
130
Q

What fluoride toothpaste should not be used in a patient with multiple porcelain crowns?

A

Acidulated

How well did you know this?
1
Not at all
2
3
4
5
Perfectly
131
Q

What’s the concentration of acidulated phosphate fluoride is used in the dental office?

A

1.23%

How well did you know this?
1
Not at all
2
3
4
5
Perfectly
132
Q

Dentist places sodium fluoride on patient with GI fillings rather than acidulated fluoride because

A

– acid of fluoride will wear away

at GI.

How well did you know this?
1
Not at all
2
3
4
5
Perfectly
133
Q

What fluoride tx would you used in a pt with amalgams, PFM’s, composite restorations, implants?

A

1.1% NaF

How well did you know this?
1
Not at all
2
3
4
5
Perfectly
134
Q

Which one is not useful in managing caries in elderly?

A

Use of 1.1% fluoride as a standard of care

How well did you know this?
1
Not at all
2
3
4
5
Perfectly
135
Q

2.2 mg NaF how much F-

A

1 mg

How well did you know this?
1
Not at all
2
3
4
5
Perfectly
136
Q

A 2 y/o child has injested 20mg fluoride pill. What will likely happen? coma, nausea, renal failure, cardiac arrest

A

nausea

How well did you know this?
1
Not at all
2
3
4
5
Perfectly
137
Q

How much fluoridation supplement would you give to a 5 y/o drinking 0.75ppm F in their water?

A

0 ppm

How well did you know this?
1
Not at all
2
3
4
5
Perfectly
138
Q

3 y/o patient lives in area with 0.4 ppm fluoride. How much do you supplement?

A

0 ppm

How well did you know this?
1
Not at all
2
3
4
5
Perfectly
139
Q
4 yrs old patient lives in community w/ 0.25 ppm fluoride intake, what do you give?
Give her systemic fluoride (0.5 mg/day)
Apply fluoride
Change diet to more fluoride intake
Prescription fluoride rinse
A

Give her systemic fluoride (0.5 mg/day)

How well did you know this?
1
Not at all
2
3
4
5
Perfectly
140
Q

4 y/o pt with 0.4 ppm fluoride Supplement?

A

0.25mg/day

How well did you know this?
1
Not at all
2
3
4
5
Perfectly
141
Q

4.5 y/o child with 0.75ppm fluoride in their water requires how much fluoride to be supplement?

A

0 mg

How well did you know this?
1
Not at all
2
3
4
5
Perfectly
142
Q

7-year-old patient has no fluoride in drinking water. What do you give them systemically…? 5 mg, 1 mg, 0.25 mg

A

1 mg

How well did you know this?
1
Not at all
2
3
4
5
Perfectly
143
Q

7-year-old child lives in area with 0.2 ppm fluoridated water, what do you supplement?

A

1.0 mg/day

How well did you know this?
1
Not at all
2
3
4
5
Perfectly
144
Q

Supplementation for 10-year-old with no other fluoride source? 1 mg/day or 1 mg/week?

A

1 mg/day

How well did you know this?
1
Not at all
2
3
4
5
Perfectly
145
Q

Diabetes, can you place implant if HbA1c = 8:

A

No, refer to physician

How well did you know this?
1
Not at all
2
3
4
5
Perfectly
146
Q

Pt with hemoglobin A1C of 12%. Pt just visited the MD, what kind of TX we can do?

A

Consult with an MD prior to tx

How well did you know this?
1
Not at all
2
3
4
5
Perfectly
147
Q

Time management of diabetic patient:

A

Treat diabetic patient 2 hours after eating & taking insulin.

How well did you know this?
1
Not at all
2
3
4
5
Perfectly
148
Q

Kidney dialysis: best to do tx when

A

day after dialysis or inbtwn days of dialysis

How well did you know this?
1
Not at all
2
3
4
5
Perfectly
149
Q
A

give OJ

Do NOT give more insulin, blood sugar is already low enough. Give OJ.

How well did you know this?
1
Not at all
2
3
4
5
Perfectly
150
Q

Pt is a child and is diabetic undergoes hypoglycemia in the chair

A

if conscious give him orange juice (unconscious give him 50%

dextrose IV)

How well did you know this?
1
Not at all
2
3
4
5
Perfectly
151
Q

Child goes into insulin shock in the chair (hypoglycemia)

a. gives OJ
b. ask parent to give kid insulin shot

A

gives OJ

How well did you know this?
1
Not at all
2
3
4
5
Perfectly
152
Q

Unconscious diabetic is treated with:

A

50% dextrose in IV.

How well did you know this?
1
Not at all
2
3
4
5
Perfectly
153
Q

IV sedation for DIABETICS:

A

Schedule in the morning.

  • If insulin-dependent, have them not eat, not take short acting insulin and take half dose of long acting insulin.
  • If not dependent, no food and no meds until after the surgery.
How well did you know this?
1
Not at all
2
3
4
5
Perfectly
154
Q

Patient is a non-insulin dependent diabetic & needs minor oral surgery w/ IV sedation. What should he do?

A

clear-liquids and regular
dose of diabetes meds.
- Minor surgery: normal as long as procedure occurs within 2 hours of eating and taking meds.

How well did you know this?
1
Not at all
2
3
4
5
Perfectly
155
Q

Pt with insulin dependent diabetes & having sedation IV and LA. Ask the pt to take: high calorie food with insulin, low calorie food
with insulin

A
low calorie food
with insulin (reduce dose of insulin and no food)
How well did you know this?
1
Not at all
2
3
4
5
Perfectly
156
Q

5 principles of ethics

A

Beneficence- “Do good”.

  • Patient Autonomy- “Self-governance”.
  • Nonmaleficence- “Do no harm”. Dentists are to keep skills and knowledge up-to-date and practice within their limits in order to protect
    the pt from harm.
  • Justice- “Fairness”.
  • Veracity- “Truthfulness”.
How well did you know this?
1
Not at all
2
3
4
5
Perfectly
157
Q

Pt presents with amalgam restorations in good shape and the dentist suggest to change them for composites due to systemic
toxicity of the amalgam. What ethic principle is the dentist is violating?

A

Veracity

How well did you know this?
1
Not at all
2
3
4
5
Perfectly
158
Q

What principle has to do with a patient’s self-governance & privacy?

A

Autonomy

How well did you know this?
1
Not at all
2
3
4
5
Perfectly
159
Q

Dentist keeps on current dental medicine to provide current standard of care. What part of the ethical code does this relate to?

A

Non-maleficence

How well did you know this?
1
Not at all
2
3
4
5
Perfectly
160
Q

Dentist refers a difficult case to a specialist, what ethic principle is this?

A

Non maleficence

- Having non-maleficence is knowing your limitations and referring patients out to specialist

How well did you know this?
1
Not at all
2
3
4
5
Perfectly
161
Q

QUESTION: Informed consent which ethics principle

A

à autonomy*

How well did you know this?
1
Not at all
2
3
4
5
Perfectly
162
Q

Dentist lets the patient sign informed consent -

A

autonomy

How well did you know this?
1
Not at all
2
3
4
5
Perfectly
163
Q

What you do first before getting informed consent? make sure patient can sign or has guardian, consult physician, discuss options
with relatives, etc

A

make sure patient can sign or has guardian,

How well did you know this?
1
Not at all
2
3
4
5
Perfectly
164
Q

90-year-old patient comes in with son, who has a document mentioning the guardian of the patient -

A

must have consent from them

to treat the patient

How well did you know this?
1
Not at all
2
3
4
5
Perfectly
165
Q

QUESTION: The 16 yr. old can take the decisions for the elder pts if: If the elders are deaf and dumb, if the boy makes the payment, if the
elders are over 60yrs, if the kid has the power of an attorney

A

if the kid has the power of an attorney

How well did you know this?
1
Not at all
2
3
4
5
Perfectly
166
Q

When should patient sign informed consent forms for surgery?

A

AFTER there has been a discussion w/ the dentist about the

surgery

How well did you know this?
1
Not at all
2
3
4
5
Perfectly
167
Q

Inform consent most contain all except:

A

cost of Tx

How well did you know this?
1
Not at all
2
3
4
5
Perfectly
168
Q

QUESTION: If you don’t obtain informed consent, what kind of offense is this?

A

Battery

How well did you know this?
1
Not at all
2
3
4
5
Perfectly
169
Q

What happen when patient doesn’t sign the consent?

A

Battery
- health care provider commits a battery if the provider performs a procedure for which the patient has not given consent.

How well did you know this?
1
Not at all
2
3
4
5
Perfectly
170
Q

Emancipated minor: if the kid is under 18, know exceptions of how they become emancipated minor

A
  • Emancipated minor assumes most adult responsibilities before reaching the age of majority (usually 18). If she/he graduated from
    high school, has been married, has been pregnant, or responsible for his or her own welfare and is living independently of parental
    control and support.
How well did you know this?
1
Not at all
2
3
4
5
Perfectly
171
Q

How is FACT witness is different from expert specialist?

A
  • fact witness = individual, who has personal knowledge of events pertaining to the case & can testify as to things they have
    personally observed or witnessed. They may not offer opinions, which are the province of the expert witness
  • Expert witnesses offer opinions, unlike a fact witness, that may assist the judge in understanding technical knowledge in order to
    support their ability to make a sound ruling in a case. An expert witness can be a credentialed specialist in fields.
How well did you know this?
1
Not at all
2
3
4
5
Perfectly
172
Q

Child came with a history of aggressive behavior and is crying, then should the dentist show empathy or sympathy or control

A

sympathy

How well did you know this?
1
Not at all
2
3
4
5
Perfectly
173
Q

Rapport best with: empathy; sympathy, compassion

A

empathy

How well did you know this?
1
Not at all
2
3
4
5
Perfectly
174
Q

What best characterizes rapport?

A

Understanding patient’s feeling and talking with patient

How well did you know this?
1
Not at all
2
3
4
5
Perfectly
175
Q

Definition of rapport?

A

mutual openness / harmonious relationship

- Rapport = mutual sense of trust and openness between individuals that, if neglected, compromises communication.

How well did you know this?
1
Not at all
2
3
4
5
Perfectly
176
Q

A successful practice is built on -

A

Good clinician-patient relationship

177
Q

Empathy is not: shared personal experiences, Imagination, understanding

A

shared personal experiences

178
Q

Which do you not need to show empathy to the patient?

a. open-mindedness
b. sharing personal experiences
c. reflection and showing understanding

A

b. sharing personal experiences

179
Q

When should the dentist NOT use para-phrasing?

a. When trying to speak to a patient in his second language
b. When the dentist is upset with what patient says
c. when giving factual values

A

c. when giving factual values

180
Q

Finding out whether a pt is listening:

A

Eye contact

181
Q

Patient complains of pain in relation to a particular tooth. The best answer/reply of the dentist would be:
If you came here earlier things would not be bad
If you took more care this would not have happened
I will take care of everything

A

I will take care of everything

182
Q

While the dentist is preparing a large carious lesion in Tooth #30 for a restoration, a pulp exposure occurs. The patient angrily
shouts at the dentist, “Your incompetent ‘creep’- -you’re responsible for this problem!”- Of the following possible responses the dentist could
make, which one is the most emphatic?
A. Calm down, I can still restore your tooth adequately.
B. Not when I’m preparing a tooth with caries like you had.
C. I can see that you’re very upset. You thought the tooth could be restored and now this problem has occurred.
D. If you took care of your mouth the way you should, I wouldn’t have been close to the pulp.
E. I’m sorry this happened, but we must get on with the procedure.

A

C. I can see that you’re very upset. You thought the tooth could be restored and now this problem has occurred.

183
Q

When the dentist enters the operatory, the patient, who is new to the office, stands close to the wall, has his arms folded, and is
looking at the floor. The dentist should initiate communication by saying which of the following?
A. Let’s get going; I’ve got a lot to do.
B. What are you angry about?
C. Didn’t my assistant get you seated?
D. You seem uncomfortable; did you have a bad dental experience?
E. Hi, I’m Doctor Wilson, what brings you here today?

A

Hi, I’m Doctor Wilson, what brings you here today?

184
Q

Pt complains of high fees of dentist, how should the dentist answer? Fee is fine according to the geographic area, it is fair and
reasonable, I have to make a living too

A

Fee is fine according to the geographic area

185
Q

QUESTION: The closest a dentist should get to their patient is?

A

Tap their shoulder

186
Q

Reason to not have parent in room with dentist and kid? communication barrier between dentist and child, OSHA violation, HIPAA
violation,

A

communication barrier between dentist and child

187
Q

Precontemplation stage of change

A

are not even thinking about changing their drinking behavior. They may not see it as a problem, or they
think that others who point out the problem are exaggerating.

188
Q

Contenplation:

A

Individuals in this stage of change are willing to consider the possibility that they have a problem, and the possibility offers hope
for change. However, people who are contemplating change are often highly ambivalent. They are on the fence. Contemplation is not a
commitment, not a decision to change.

189
Q

Pt. says, “I do not have time to quit smoking.” What stage is s/he in?
A: Precontemplation, contemplation, action, denial

A

Precontemplation

190
Q

over the counter nicotine replacement therapy

A

patches, gum, lozenges

inhaler and spray are prescription

191
Q

buproprion uses

A

aka Zyban, Wellbutrin

reduce nictoine withdrawal, also depression
can be used safely with nicotine replacement

192
Q

buproprion contraindications

A

seizure disorder, current or prior diagnosis of anorexia nervosa or bulimia, or
undergoing abrupt discontinuation of alcohol, benzodiazepines, barbiturates, and antiepileptic drugs.

193
Q

varenicline use

A

aka chantix

reduce nictoine withdrawal, blocks effect of nicotine if starts smoking again

194
Q

varenicline contraidications

A

Alcohol Intoxication, Depression, Increased Cardiovascular Event Risk, Coronary Artery Bypass Graft, Heart Attack,
Acute Syndrome of the Heart, Angina, Transient Ischemic Attack, Stroke, Peripheral Vascular Disease, Severe Renal
Impairment, Seizures, Feel Like Throwing Up, Schizophrenia, Manic-Depression, Having Thoughts of Suicide, Habit of
Drinking Too Much Alcohol. DON’T COMBINE WITH PATCHES

195
Q

which non-nictoine replacement can be combined with nictoine replacement

A

buproprion only

NOT VARENICLINE

196
Q

positive reinforcement

A

Positive consequence that increases behavior

§ you brush, you get sticker

197
Q

negative reinforcement

A

Removal of negative consequence that increases behavior

§ Stop pain from toothache, pt realizes he should brush

198
Q

positive punishment

A

aversive Conditioning, negative consequence that decreases negative behavior

§ everytime you don’t brush, you have to clean your room

199
Q

negative punishment

A

Removal of positive stimulus in order to decrease an undesirable behavior

§ don’t brush, no allowance

200
Q

operant extinction

A

child cries, don’t give attention

201
Q

what is more effective, reinforcement or punishment

A

reinforcement is more effective than punishment because in punishment, you have resentment, you avoid the punisher,
and you are not taught positive behavior.

202
Q

Most important component of systematic desensitization is

A

exposure to fearful stimulus

203
Q

During the child’s first visit, the dentist requested that the parents wait in the reception room. The child cries moderately, but
tearfully, throughout the dental examination and prophylaxis. The dentist “gave her permission” to cry while he/she worked and then took no
notice of her crying. Her crying diminished in intensity over time and then stopped. With respect ONLY to the crying behavior, the dentist has
A. used positive reinforcement.
B. used negative reinforcement.
C. extinguished the behavior.
D. ignored the problem.

A

C. extinguished the behavior.

204
Q

Definition of Operant extinction?

A

removal of reinforcements that decrease a behavior

- disappearance of a previously learned behavior when the behavior is not reinforced.

205
Q

Behavior shaping:

A

providing positive reinforcement for approximation of behavior you are desiring

206
Q

A behavior modification device (ie thumb sucking deterrent) is an example of:

A
POSITIVE PUNISHMENT (negative consequence that
decreases negative behavior)
207
Q

Patient is given oral habit reducing appliance to prevent an oral habit, what is this considered?

A

POSITIVE PUNISHMENT

208
Q

Providing reward for desired behavior:

A

positive reinforcement

209
Q

Eye contact, smiling, and telling pt doing good job (praising) is what type?

A

social reinforcement

210
Q

Desensitization works if the base of the behavior problem is

A

fear

211
Q

Main disadvantage to desensitization?

A

Time

212
Q

classic conditioning in how a patient sees a dentist

A

sees dentist (stimulus) – assumes pain is coming

213
Q

Conditioned stimulus?

a. Dental chair
b. High blood pressure
c. Fear
d. Anxiety

A

a. Dental chair

214
Q

How to deal with angry patient? Listen and validate emotion, agree with patient, ignore anger then investigate after

A

Listen and validate emotion

215
Q

How to reduce stress & dental anxiety in peds?

A

Tell-show-do

216
Q

Based on Frank behavioral rating scale, what is the rate that indicates positive rapport with dentist?

A

rating 4

217
Q

6-year-old int. disabled child. Treatment is a recall. Would you give sedation, antianxiolytic, voice control or positive
reinforcement?

A

positive
reinforcement?
- with ID, you want to be short and brief, explain things, tell-show-do, and REWARD. Positive reinforcement.

218
Q

What is the best way to treat a developmentally disabled patient?

A

Consistency

- Disabled pt, should be treated by flatterness, permissible, consistency

219
Q

Autistic kids have what characteristic?

A

Repetitive behavior

220
Q

Disable patient comes in and not cooperative, how should you act?

A

Permissiveness (give patient freedom & treat in the way patient
feel comfortable)

221
Q

Patients with autism will usually show?

a. decreased rate of caries
b. heightened sense of lights and sounds
c. the compassion to interact with people

A

b. heightened sense of lights and sounds

  • Children with autism are easily overwhelmed by sensory overload, which can cause “stimming” (flapping of arms, rocking,
    screaming, etc).
  • Autistic children are hypersensitive to loud noises, sudden movement, and things that are felt.
222
Q

Child patient – you smile, tell him good job, and pat him on the shoulder. These are examples of negative reinforcement, social
reinforcement, or token reinforcement.

A

social

reinforcement

223
Q

If kid complained and whined in the beginning but at the end, is very good: you

A

compliment how well they were at the end of the

procedure.

224
Q

QUESTION: Voice control method used with children’s -

A

Aversive conditioning (punishment to deter unwanted behavior, ex. hand over mouth)

225
Q

What is the purpose of the voice control technique?

A

Sets boundaries à Aversive conditioning

226
Q

QUESTION: 8-year-old patient, 1st dental visit ever, scared of dentist? What’s the most likely answer?

a. Television
b. Parents
c. Peers

A

b. Parents

227
Q

If a child is afraid,

A

allow the child to express fears

228
Q

How do you treat a fearful child? use sedation, let him watch another patient,

A

let him watch another patient,

229
Q

Young patient is scared b/c he has no control what to do?

A

tell him to raise his hand if he needs a break/ you to stop

230
Q

A kid is on recall appointment and is not cooperative. You should do voice control followed by?

A

v

231
Q

Patient is very young and fearful first time you meet them –

A

try to talk to them, go down at their height.

232
Q

Patient 2 yrs old and scared, who do you ask to help position the patient?
ask parent to position patient for you
get assistant to do it
you do it yourself

A

ask parent to position patient for you

233
Q

The restraining of uncooperative 2 yr. child should be done by. Dentist, Assistant, Parent

A

Parent

234
Q

4-year-old kid, best position?
Knee to knee with head on dentist lap
Knee to knee with head on parent’s lap

A

Knee to knee with head on dentist lap

235
Q

Patient comes in with 1-year-old child, how do you do exam?

A

parent and dentist are knee to knee, baby’s head is in dentist’s lap

236
Q

8-year-old boy, when will he behave better?
Mom inside the dental office
Dad inside the dental office
Nobody inside

A

Nobody inside

237
Q

What is a 2 yr. old most afraid of? 4 yr. old?

A

• 1-3 yr. old: SEPARATION
• 4-6 yr. old: UNKNOWN
- pediatric fears correlated with age

238
Q

QUESTION: Uncooperative 2-year-old, what are they scared of?

A

separation anxiety

239
Q

4-5-year boy is scared of?

A

Unknown

240
Q

You help a child help recognize what they are afraid of and make outward positive connection:

A
cognitive restructuring
(psychotherapeutic process of learning to identify and dispute irrational or maladaptive thoughts)
241
Q

Behavior Modeling:

A

when the kid is afraid and you use a sibling or someone older to show how they should behave

242
Q

QUESTION: A 4 yr. old child management?

A

empathy and respect

243
Q

Replacing words like LA with sleepy juice is called as

A

Euphemism (relabeling)

244
Q
Pedo 1st visit. Multiple carious teeth on anteriors. During anesthesia is well cooperative and doesn’t cry or move. Once begin tx,
begins to cry. What do.
• Keep working
• Voice control
• More anesthesia
• Oral sed
• N20
A

• Voice control

245
Q

Fear vs anxiety:

A

Fear: results from anticipation of a threat arising from an external origin.
Anxiety: results from anticipation of a threat arising from an unknown or unrecognized origin.
- Anxious patients are the most difficult patients as they often cause the dentist to become anxious as well.

246
Q

fear increases or decreases pain?

A

increases

247
Q

Dental anxiety can be caused by patient’s helplessness. What would reduce it?

A

Telling the patient to raise her/his hand when feels

pain

248
Q
A patient is going to the dentist and has never had local anesthetic. He recently got a flu vaccine and is now afraid of needles. The
fear is due to what?
Location
Generalization
Translation
A

Generalization

249
Q

Define anxiety according to Freud and K:

A

aversive inner state that people seek to avoid or escape.

250
Q

What do Freud and Erikson say about anxiety?

A

Inability to overcome a conflict in a particular stage that will lead to anxiety.
Inadequate resolution becomes anxiety
- An inadequate resolution would indicate a child’s insecurity and anxiety.
- An Adequate Resolution would mean that a child was able to overcome the conflict in each stage and develop properly. This
applies similarly to the other 8 stages.

251
Q

Patient has dental fear, what is most likely due to?

A

previous traumatic dental procedure

252
Q

What would most cause a man to have anxiety? traumatic past experience, or finances, peers, unpleasant staff

A

traumatic past experience

253
Q

Constantly exposing the pt to get from the fear factor is .

A

desensitation

254
Q

Systematic desensitization- 3 steps:

A

Construct a hierarchy, relaxation exercises, associate components of hierarchy with relaxation
state

255
Q

Panic attacks usually produce a sense of unreality, a fear of impending doom, or a fear of losing control.

A

a fear of impending doom

256
Q
What is not included in the ADA code of ethics?
• Licensure by credential
• Advertising
• patient values
• Fees
A

• Fees

257
Q

What cannot be advertised by a general dentist?

a. Cost
b. Specialty
c. License agreement

A

What cannot be advertised by a general dentist?

a. Cost
b. Specialty
c. License agreement

258
Q

A dentist has an ethical obligation to report a colleague in all these situations except?

a. working under substance abuse
b. advertising on electronic media
c. abusing patients

A

b. advertising on electronic media

259
Q

if you find problems with a medical conditions occurring with a certain drug, who do you contact? OSHA, FDA, EPA

A

FDA

260
Q

If there is an adverse reaction to a medication in the office, who do you notify? a) FDA b) CDC c) HIPPA d) OSHA e) EPA.

A

FDA

261
Q

Asked which statement was correct for HIPPA?

A

Must give privacy form to pt but you don’t need confirmation of receipt, fax and
email standard, etc.

262
Q

What’s not the reason for rising dental costs?

A

number of dental students in dental schools

263
Q

medicaid vs medicare

A

medicaid is income based

medicare is age-based (>65 yo)

264
Q

insurance pays a flat fee - patient pays the rest

A

co-pay

265
Q

provider paid per patient not per procedure

A

capitation

266
Q

HMO

A

limited to selection
– dentists are paid a fixed rate for each individual per month. Dentist is paid regardless patient was seen or not. If value of
services exceeds payments, dentist’s loss. If payment exceeds value of services, dentists gain.

267
Q

PPO

A

group of dentists agree on certain lower fees in hopes o attract more patients

268
Q

who decides how dentists are reimbursed

A

each state has discretion

269
Q

QUESTION: What does Medicaid cover?

A

Extractions, one-time denture, children until 18.

270
Q

What sector of government provides funding for dental care? Medicaid, Medicare, grant, HMO

A

Medicaid

271
Q

Who pays for Medicare:

A

federal program that pays for covered health services for most people 65 years old and older and for
most permanently disabled individuals under the age of 65.

272
Q

Government spends most of the money in Medicare, Medicaid, HMO.

A

Medicare

273
Q

Medicare is a federal program that provide health care for elderly. It does not cover dental.

A

Answer: Both statements

are true

274
Q

Most aid for finance: Medicaid, Medicare, and hmo

A

Medicaid

275
Q

Most dental procedures for the elderly are paid for by

A

out of pocket cash.

276
Q

Which of the following is the leading payer for dental treatment? Insurance or self-pay

A

self-pay

277
Q

Who pay for most of dental care?

a. government
b. insurance
c. cash

A

cash

278
Q

Majority of health service in USA:

A

private insurance.

279
Q

QUESTION: who pays most of dental Tx:

A

56% patients, 33% third parties private insurance

280
Q

73yo woman makes $23,000/year. how should she receive dental care?

  • Medicaid
  • Medicare
  • Private insurance
A

• Private insurance

281
Q

A 65 yr. old lady living on 40k pension per year, wants to get dental treatment. She does not have any other physical abnormality
besides tooth pain in her molars. From where does the money covered for her treatment come from?
a. Medicaid
b. Medicare.
c. Private Insurance
d. Others insurance.

A

c. Private Insurance - private dental IF she has it\

medicaid does not cover dental for adults
medicare does not cover dental for elders

282
Q

What is the name of the federal funded medical care for the elderly and its coverage?

a. Medicare wI dental coverage
b. Medicare w/o dental coverage
c. Medicaid wI dental coverage
d. Medicaid w/o dental coverage

A

b. Medicare w/o dental coverage

283
Q
Insurance question about adverse selection 
• only take pt with high risk
• only take pt with low risk
• take both
• something about taking pt of all ages
A

(adverse selection deals with the idea that those at higher risk are more likely to buy an
insurance policy. If the price for the policy is the same for nonsmokers and smokers, it is more likely that smokers will buy the insurance,
because it is more “worth it” to them—because they are at higher risk for disease. This is averse to the insurance. So the prices need to be
different.

• only take pt with high risk

284
Q

Know about capitation:

A

Dentist is paid a fixed fee to see patients enrolled in program; HMO = capitation dental plan

285
Q

which is the capitation dental plan

A

HMO

286
Q

most common dental plan

A

HMO

287
Q

who’s usually at risk with HMO

A

dentist

288
Q

Insurance allows pt to only see certain set of providers…. PPO, HMO, Closed panel

A

Closed panel

289
Q

Company offers dental insurance to its employees that can go to selected dentist, what is this example of?

A

Closed panel

290
Q
On a prepayment basis, dental patients receive care at specified facilities from a limited number of dentists. This practice plan is
classified as which of the following?
A. Closed panel
B. Open panel
C. Group practice
D. Solo practice
A

A. Closed panel

291
Q

Direct Reimbursement:

A

self-funded group dental plan in which the employee is reimbursed based on a percentage of dollars spent
for dental care provided, and which allows employees to seek treatment from the dentist of their choice.

292
Q

If you are an employer and you provide your employee with reimbursements for dental care they received from a dentist of their
choice it is called:

A

direct reimbursement,

293
Q

patient goes to the dentist and needs to pay something before seen:

A

Copayment

294
Q

What happened in 1997:

A

SCHIP (state children insurance health program)
- SCHIP (State Children’s Health Insurance Program) provides matching funds to states for health insurance to families with children.
It covers uninsured children in families with incomes that are modest but too high to qualify for Medicaid.

295
Q

1997 law passed that

A

state must look after children that cannot afford healthcare - State Children’s Health Insurance
Program (SCHIP) AKA Children’s Health Insurance Program (CHIP)

296
Q

Unbundling -

A

separating of a dental procedure into component parts with each part having a charge so that the cumulative charge of the
components is greater than the total charge to patients who are not beneficiaries of a dental benefit plan for the same procedure.

297
Q

Bundling -

A

opposite of unbundling & can occur on the insurance carrier end. It’s the systematic combining of distinct dental procedures by
third-party payers that results in a reduced benefit for the patient/beneficiary.

298
Q

Upcoding or overcoding is defined by the ADA as

A

“reporting a more complex and/or higher cost procedure than was actually performed.”

299
Q

Downcoding is defined by the ADA as

A

“a practice of third-party payers in which the benefit code has been changed to a less complex and/or
lower cost procedure than was reported except where delineated in contract agreements.”

300
Q

Dentist did not accept a copayment and did not report it to the 3rd party (insurance)?

A

Overbilling

301
Q

Dentist charge for $500 for a crown. insurance only covers $400. Dentist waves copayment ($100) but still let insurance knows that
he charges $500 for crown, what’s this action called?
a. Down coding
b. Overbilling
c.Price fixing
d.Unbundling

A

Overbilling

302
Q

The dentist charges separately for core build up and the crown but the insurance company says that the core builds up is part of
crown. What is this called?

A

Bundling

303
Q

Doctor billed insurance couple of procedures, when actually there is a global procedure that combines them all, what did he
commit?

A

Unbundling

304
Q

Dentist does the treatment for 2 crowns but the insurance company paid for one crown, what is it?

A

Downcoding

305
Q

You performed a two surface restoration and coded it that way. Insurance came back with coding it as only one surface restoration.
What is this called? Downcoding or upcoding

A

Downcoding

306
Q

What is it called when a dentist charges several procedures instead of one?

a. upcoding
b. downcoding
c. unbundling
d. bundling

A

unbundling

307
Q

The patient retires & loses health benefits. The treatment is done on the next day. The pt requests that the
dentist enter the previous day’s date and the dentist does so. What is this called?

A

Fraud

308
Q

Child abuse sign:

A
  • multiple untreated injuries
  • lag time bt injury and tx
  • comminuted facial fractures
  • parents with different stories
  • Most common in children under 3
309
Q

abuse most common in which children

A

<3

310
Q

Abuses that have to be reported to authorities -

A

colleague practicing with chemical impairment, colleague advertising falsely on media, child
abuse, domestic violence, elderly abuse

311
Q

When treating elderly patients what should be your concern?

A

Health of patient

312
Q

How often will you see child abuse

A

at least twice a year

313
Q

It is required mandatory to report all except: child abuse, reaction to drug

A

reaction to drug

314
Q

You suspect child abuse. Who do you call?

A

Social services

315
Q

If there is an elderly woman in your chair & you think there might be abuse. What do you have to do? Tell family or tell human
health services

A

tell human

health services

316
Q

Which is not true of elder abuse:
Most of the elder abuse is at victims home
Mostly it is by the victim’s relative
elder’s abuse is often over reported and exaggerated

A

elder’s abuse is often over reported and exaggerated

317
Q

Unauthorized use of elderly’s ATM card is not a sign of abuse but in some situation, it is under consideration.

A

(Both are true)

318
Q

Elderly abuse is often:

A

underreported

319
Q

Dentist potential for abuse not likely due to

a. Vulnerability
b. Pressure of being perfect
c. Knowledge and access to drugs
d. Stress

A

Vulnerability

320
Q

When opening a dental practice, what makes it more successful?

A

Better communication

321
Q

What do general dentists report as being their biggest issue? fearful patients, business/financial issues, staff training

A

fearful patients

322
Q

Patient is bothering the dentist, dentist got upset. The assistant drops instruments in the floor & the dentist was so piss that he had
it out with the assistant. What you you call that reaction ?

A

Transference

- Transference is a unconscious redirection of feelings from one person to another

323
Q

Most eye injury in practice happens to who: dentist, dental assistant, hygienist, custodian

A

dentist

324
Q

Least chance of needle injury? Setting up, Cleaning up, Recap

A

Setting up

325
Q

When do you most likely get a puncture wound: pre procedure, during, post-proceduring cleanup, needle recapping

A

post-proceduring cleanup,

326
Q

Most injury/percutaneous cuts happen when

A

recapping needles

327
Q

What test for every year?

A

Hep B

328
Q

Dentist can diagnose which of the following?

A

Bulimia (reflected in oral condition)

329
Q

A patient comes in with rampant decay. What is the primary responsibility of the dentist?

A

figure out etiology of decay FIRST

330
Q

Patient is in your office for a treatment plan, all of the following should be done when you explain the proposed treatment to the
patient, except? Use professional terminology explain risk of not getting a procedure done, inform on the fee of the procedure

A

Use professional terminology

331
Q

QUESTION: First step before/in treatment planning: make sure patient doesn’t need translator, consult with physician about pre-existing
medical conditions

A

make sure patient doesn’t need translator

332
Q

Patient is ready to hear your treatment plan, all of the following are true except?

A

Guarantee the success of treatment!

333
Q

First step in tx planning is?

A

treat the initial pain and discomfort of the pt. Other choices were see how you can make a preventitive
plan, treat all restorations.

334
Q

Proper order for treatment planning –

A

emergency care, disease control, reevaluation, definitive treatment, maintenance care

335
Q

Which are the two most imp. steps for diagnosis:

A

History and clinical examination

336
Q

Patient comes to your office, complains about how other dentists did really bad job, and tells you how you are the best dentist in
the world. What mental condition is she suffering from?

A

Borderline Personality Disorder

337
Q

Pt comes in saying she’s been to 5 different dentists the last 6 months. A few minutes later, she’s telling you how great of a dentist
you are and that she’ll refer all of her friends to you. This example of:

A

Borderline. Other choices were paranoia & schizoid.
- Borderline personality disorder (BPD) is a serious mental illness marked by unstable moods, behavior, and relationships.

338
Q

Patient with bipolar disease comes in for dental care, choses not to take his medication and states he is in the “manic phase,” what
do you expect from treating this patient?: he will have unpredictable reactions during the treatment, he is will be obsessed about is esthetics

A

he will have unpredictable reactions during the treatment,

339
Q

When trying to change person, what is most important?

A

trying to determine whether they are willing to change

340
Q

Patient who has a complex medical history that is not debilitating but will require medical management and dental modifications –

A

ASA 3

341
Q

You have a test that is not accurate but gives consistent result: this means test is

A

reliable

342
Q

Which of the following are necessary for a test to be accurate: Specificity, reliability, validity

A

validity

343
Q

is verbal or nonverbal communication more reliable

A

verbal

344
Q

Dentists have to have proper accommodations for disable people. Dentists have to treat HIV people the same as others.

A

Both

statements are true

345
Q

American disabilities act does not include HIV:

A

False

346
Q

One patient left, and before getting another patient, how would you clean your operatory?

A

Use disinfecting spray à let it sit for 10

minutes and then wipe off

347
Q

Disinfection?

A

Destroy majority of microorganisms but not bacterial spores.

- Bacterial spores = benchmark organism for sterilization

348
Q

QUESTION: Benchmark for sterilization:

A

bacillus spores

- I think it’s Clostriduim Botulinium

349
Q

Mycobacterium is the benchmark for

A

disinfection

350
Q

Definition of disinfectants –

A

Inanimate objects (non-living)

351
Q

Antiseptic: can be safely applied to

A

tissues, but will kill most living organisms

352
Q

Denaturation of the proteins - _______; Coagulation of proteins - ________

A

QUESTION: Denaturation of the proteins - alcohol and autoclave; Coagulation of proteins - dry heat

353
Q

QUESTION: Steam Autoclave:

A

20 minutes at 1210C & 15 psi.

354
Q

Which method of sterilization needs higher temperature:
steam
dry heat
oxide pressure

A

dry heat – 160 C or 3200 F

355
Q

Temperatures for autoclaves is governed by:

A

FDA

356
Q

Which method of sterilization does not corrode instruments/burs? – Dry Heat, Ethylene oxide

A

Dry Heat

357
Q

Which method of sterilization does not dull carbide instruments?

A

Dry heat

358
Q

Sterilization most destructive to burs & causes rusts: Steam heat, dry heat, unsaturated vapor, chemical, ethylene oxide

A

Steam heat

359
Q

What is best to sterlize carbide burs?

A

DRY HEAT or unsaturated chemical vapor –> no corrode or dull
- Ethylene oxide is for heat-sensitive instruments

360
Q

Anti-retraction valves -

A

used to prevent aspiration of patient materials into some dental handpieces and waterlines – prevent
patient to patient cross-contamination.

361
Q

What’s not found on the OSHA poster?

A

How many days each employee is allowed to work with that chemicals

362
Q

How often should OSHA be updated

A

once a year

363
Q

What are the hep B vaccine rules by OSHA?

A

all must always be offered and able to get the vaccine

364
Q

Once a year, you have to check for one of the following:
HIV
HEP B
HEP C

A

HEP B

365
Q

If worker didn’t get Hep B vaccine because he is more concerned about HIV?

A

Tell him it’s easier to get hep B à must sign that they

legally don’t want

366
Q

Who is at least risk for HEP B infection?

a) food servers
b) Down syndrome
c) drugs addicts

A

a) food servers

367
Q

Who is in control of writing the material safety data sheet (MSDS):

A

Manufacturer

368
Q

Hazard Communication Standard:

A

created by OSHA to make sure employees know about hazardous/toxic materials

369
Q

Hazard Communication Standard states –

A

every chemical hazard has to be evaluated then reported to employer & employees

370
Q

HAZARD COMMUNICATION LAW created by?

what does it control: amalgam, sharps, blood

A

a) created by OSHA

controls, among others, amalgam

371
Q

hazardous communication regulation

A

train worker right after you hire; train worker when new hazardwhether the means of 2 groups are statistically different—assume that standard deviation is unknown. Small sample
sizeous material is in office

372
Q

T test: used to compare

A

whether the means of 2 groups are statistically different—assume that standard deviation is unknown. Small sample
size

373
Q

Z test: used to compare the

A

means of 2 groups are statistically different if the variances like standard deviation are known. Large sample size.

374
Q

Chi-square test:

A

tests correlation b/w two independent variables

375
Q

Case control study —

A

RETROSPECT study that compares people that have the disease to people that do not have the disease. Also, looks back
to see how the risk for the disease is compared to actually getting that disease.
- start with disease and look backwards for exposure
- ex. How did people react to the new mouthwash vs nonusers

376
Q

Cohort study —

A

PROSPECTIVE study where there is more than one sample/cohort and evaluations are done to see how certain risk factors the
groups have are related to developing a certain disease.
- look forward from exposure to disease development, through time to see how develops disease, then looks at various factors (no
one has diease yet)
- ex. how will/does people react to using new mouthwash vs nonusers

377
Q

Cross sectional study —

A

EPIDEMIOLOGICAL study that looks at the entire population. Not like case control, that only studies a certain group
with a specific characteristic.
- all variables measures simultaneously at one point in time
- Example: It was observed that there was less caries in certain geographic areas. Higher fluoride in water supplies was suspected
as the probable cause

378
Q

Longitudinal Study —

A

TIMED study that looks at a certain set of people (same people) over a long period of time.

379
Q

gold standard for establishing cause and effect

A

clinical trial

380
Q

If a dentist is reading an article, where should he look for the definition of dependent and independent variables? method ,
introduction, discussion, results, summary

A

method

381
Q
Where would you look in a scientific journal to find the dependent and independent variables
• Intro
• Materials
• Methods
• Conclusion
• Summary
A

Methods

382
Q

What section states the purpose of the research?

A

INTRO (ABSTRACT)

383
Q

Researcher wants to find incidence of oral cancer in nursing home what study?

A

Cross-sectional

384
Q

Which type of study determines relative risk ratio:

A

Cohort

385
Q

QUESTION: What parameter study lets you have a risk quotient?

A

Cohort

386
Q

Efficacy, what study would you do?

A

CASE CONTROL

387
Q

Study among smokers & nonsmokers in a period of 6 years (2000-2006) to develop disease? Cohort, cross sectional

A

Cohort

  • By: disease/non-disease: case control; by time: cohort
388
Q

What type of studylets you find causation- analytical, cross-sectional, case-control, cohort

A

analytical

389
Q

Myestena Gravis patients are involved in a study. The doctor is conducting a study and is trying to find out how many of these
patients has periodontitis. What study is he conducting?

A

Case control study

390
Q
Dentist is doing research on 5 unrelated patient with different background. He record data ......etc. Dentist is doing what kind of
research?
a. clinical trial
b. cohort
c. sectional
A

a. clinical trial

391
Q

Study group A and B give some agents for plaque control then compare which agent is more effective. Which study is that?

A

Clinical

trial

392
Q

A study is done to determine the affectiveness of a new antihistamine .To do this, 25 allergic pt‟s are assigned to one of the two
groups ,the new drug (13 pt’s) , placebo (12 pt’s) . The pts are followed for 6 months . This study is called: Cohort, Cross-sectional, Case
controlled, historical cohort, clinical trial.

A

clinical trial. ( assigned or give is the clue )

393
Q

A study is designed to determine the relationship between emotional stress and ulcers. To do this, the researchers used hospital

records of pt’s diagnosed with peptic ulcer disease and pt. diagnosed with other disorders over the period of time from july 1988 to july 1998 .
The amount of emotional stress each pt. is exposed to was determined from these records. This study is:
A) Cohort B)Cross-sectional C) Case-study D)Historical Cohort E)Clinical Trial

A

C) Case-study

394
Q

A researcher conducting a research between students self studying and those attending lectures what is the independent variable?
students participating in research, material studied, Students results, Lecture of self study

A
  • ATTENDING LECTURE or SELF STUDY (INDEPENDENT VARIABLE)

- STUDENTS RESULT (DEPENDENT VARIABLE)

395
Q

QUESTION: Analyze statistical difference between two means?

A

T-test

396
Q

Crossover study advantages:

A

influence of confounding covariates is reduced because each crossover patient serves as his or her
own control and are statistically efficient and so require fewer subjects than do non-crossover designs (even other repeated measures designs).

397
Q

Means of caries risk assessment for 3 groups: white, black, Hispanic what test do u use to compare?
A) chi square b) variance c) t-test

A

c) t-test

398
Q

How do you compare between 2 constant variables? Chi square, regression analysis

A

regression analysis

399
Q

QUESTION: Two groups of 100 ppl, gave them different foods & asked how they felt afterwards. which test to compare the 2 groups answers à

A

chi squared test

400
Q

Want to compare 2 groups of people, male and female for something, what test do you look at? Multiple regression, Chi square
Test, T-test

A

Chi square

401
Q

QUESTION: What test measures 2 nonparametric data? Chi-square, normal distrubition, spearman, wilcoxin, kruskal wallis

A

Chi-square

402
Q

Two common VARIABLE..what statistical test would you use? Chi-test, T-test, correlation analysis, or standard deviance

A

Chi-test,

403
Q

QUESTION: If you have a study of confounding variable

A

– minimize confounding variables by randomizing
- minimize confounding variables by randomizing groups, utilizing strict controls, and sound operationalization practice all
contribute to eliminating potential third variables.

404
Q

QUESTION: confounding variants

A
  • a third variable or a mediator variable, can adversely affect the relation between the independent variable
    and dependent variable. This may cause the researcher to analyze the results incorrectly. The results may show a false correlation between the
    dependent and independent variables, leading to an incorrect rejectionof the null hypothesis.
405
Q
  • Type I Error with null hypothesis-
A

rejecting the null hypothesis when it is true. This is an alpha error. Another way to say this is, to reject a null that
should be accepted.

false positive

406
Q

Type II Error with null hypothesis-

A

accepting a false null hypothesis. This is a beta error. Another way to say this is, to accept a null that should be
rejected.

false negative

407
Q

what should observed probability be to reject null hypothesis

A

<= 0.05 (5%) – outcome is judged as no effect

408
Q

when accept null hypothesos

A

when probability >5% –> accept null, results not statistically significant

409
Q

sensitivity

A

people with disease correctly classified as having disease

true positive/(true positive + false negative [these have it but not recognized])

410
Q

specificity

A

people without disease correctly classified as not having it

true negative/(true negative + false positive)

411
Q

incidence

A

new cases

412
Q

Experiment was done and error = 0.05 was the goal but when completed it was 0.01. The question asks what type of error was it?

a. Type I
b. TYPE 2
c. no error:

A

no error: Error of less or equal of 0.5 no statistical significance..

413
Q

P-significant value is equal to 0.01, your theory should be right, so you you will ______ null hypothesis

A

reject

414
Q

Type I – and Type II –

A

1 is false rejection of null hypothesis (false negative/incorrect regection) = less dangerous in terms of research

2 is false acceptance of null hypothesis (false positive/failure to regect) – less problematic bc no conclusion is made from a rejected null. But type 2
is more dangerous medically bc a patient is diagnosised as HEALTHY when they actually have the HIV.

415
Q

The power of a statistical analysis is ultimately to:

A

reject the null

416
Q

QUESTION: Incidence is when

A

number of people like to get disease in given time

417
Q

QUESTION: Dentist in his clinic notices a new diseases this is?

A

incidence

418
Q

What is the statistical measure for the total number of cases per population, regardless of time of onset?

A

prevalence

419
Q
For a population, the research divides the number of disease cases by the number of people. By so doing, this investigator will have
calculated which of the following rates?
a. incidence
b. odds ratio
c. prevalence
d. specificity
A

c. prevalence

420
Q

Specificity:

A

Proportion of truly nondiseased persons who are so identified by a screening test (measures “how good a test is at
correctly identifying nondiseased persons).

Sensitivity tests identifying diseased persons.

421
Q

Dentist finds a group of individuals are free of (do not have the) dental disease:

A

specificity

- Specificity (without disease) and sensitivity (with disease)

422
Q

If a dentist was able to correctly ID disease free patients w/ the diagnostic study, it has?

A

high specificity.

423
Q

Study says 95 out of 100 people had the disease what is lab value:

A

95% sensitivity

424
Q

A study failed to report 5 cases of caries. What is this called? 1. True Positive, 2. True Negative, 3. False Positive, 4. False Negative

A

False Negative

425
Q

QUESTION: Incidence of caries in your office this year is 300 out of 1000, last year it was 200, so what is it for this year?

A

10%

- Incidence refers to NEW cases so the answer is (300-200)/1000 = 100/1000 = 0.1

426
Q

Dentist has 300/1000 patients with periodontitis; last year only 200 had periodontitis, what is the incidence for this year:

A

10%

427
Q

mean, median, mode

A

mean (average)
median (middle number)
mode (number that shows up the most)

428
Q

Which does not describe the spread of data? median, range, variance, standard deviation, standard error

A

median

429
Q

What do you use for average Q? Mean, median, mode

A

Mean

430
Q
Which of the following represents the variability about the mean-value of a group of observations?
A. Sensitivity
B. Standard deviation
C. t-Statistic
D. Specificity
A

B. Standard deviation

431
Q

Histogram is used to show : mean, correlation of 2 variables, variance

A

variance

standard deviation

432
Q

Outliers control:

a. mean
b. median
c. mode
d. standard deviation

A

d. standard deviation

433
Q

An outlier has the biggest effect on which of the following?

A

Standard deviation

434
Q

Your office uses perio scale 1= gingivitis 2= mild perio 3= moderate/severe etc, what type of scale is this? Nominal, ordinal, ratio,
cardinal

A

ordinal

435
Q

mild, moderate, severe: what type of scale?

A

ordinal

436
Q

kelvin temperature what type of scale

A

ratio

437
Q

celsius temperature what type of ratio

A

interval

438
Q

Best scale for gingival index?

a. ratio
b. nominal
c. interval

A

b. nominal